You are on page 1of 258

[26/06, 8:49 am] +92 324 2921513: Skip to main content

Toggle navigation

Search StratOG
HomeExam preparationPart 1 MRCOGSBAs for Part 1 MRCOG online
resourceAnatomy page 2
Tutorial history
Progress 100% complete
Submit
Anatomy page 2
Assessment History
Total Attempts: 5
Highest Score: 100 %
View last results
View highest result
Retake quiz
What artery supplies the distal third of the transverse colon?

Celiac trunk artery


Inferior mesenteric artery
Left renal artery
Right renal artery
Superior mesenteric artery
What nerve supplies to the anterior one-third of the labium majus?

Dorsal branch of the pudendal nerve


Inferior rectal nerve
Ilioinguinal nerve
Obturator nerve
Pudendal nerve
What part of the spermatozoa has the highest concentration of mitochondria?

Acrosome
Flagellate tail
Middle piece
Principle piece
True head
Which testicular cells are located outside the blood/testis barrier?

Leydig
Primary spermocytes
Sertoli
Spermatids
Spermatogonia
What type of cells are sertoli cells as defined by shape?

Circular cells
Cubodial cells
Epithelia with gap junctions
Flat columnar
Tall columnar
What is the layer between the theca cells and the mural granulosa?
Basal lamina
Basement membrane
Podocysts
VEGF unregulated capillary bed
Zona pellucida
With regard to ovulation, what is the fluid filled space in a follicle called?

Antrum
Basal lamina
Blastocyst
Lacuna
Uniblastocyst
What size is a resting follicle?

0.02 mm
2 mm
10 mm
15 mm
20 mm
What size is a follicle that is ready to ovulate?

2 mm
4 mm
10 mm
15 mm
20 mm
Regarding the urinary bladder, which of the following is correct?

A glycosaminoglycan layer coats the luminal surface of the bladder mucosa


At the trigone, the epithelium is loosely adherent to the underlying muscle
The bladder wall consists of two layers of detrusor muscle
The uterovesical fold creates the Pouch of Douglas
The urinary bladder is covered entirely by peritoneum
The internal pudendal artery leaves the pelvis via what structure?

Greater sciatic foramen


Lesser sciatic foramen
Obturator foramen
Pudendal canal
Sacral canal
A direct inguinal hernia that develops from the weakness of the conjoint
tendon can cause damage to what nerve?

Genitofemoral nerve
Iliohypogastric nerve
Ilioinguinal nerve
Pudendal nerve
Subcostal nerve
Lymphatic drainage of the uterus is primarily to:

Common iliac nodes


External iliac nodes
Internal iliac nodes
Para-aortic nodes
Superficial inguinal nodes
From which embryological structure does the round ligament of the uterus
originate?

Cloaca
Gubernaculum
Mullerian duct
Urachus
Wolffian duct
Which of the following is NOT a branch of the anterior division of the internal
iliac artery?

Ilieolumbar artery
Internal pudendal artery
Obturator artery
Uterine artery
Vaginal artery
This is the last question of this quiz. Press finish to submit your choice(s) and
reveal the answer(s).

Finish
Add/Edit reflective notes
< Previous pageNext page >
Royal College of Obstetricians and Gynaecologists
© 2018
Registered charity no. 213280
27 Sussex Place
Regent's Park
London NW1 4RG
UK

Tel +44 20 7772 6200


Fax +44 20 7723 0575
Contact Us
Accessibility
Privacy & cookie policy
Terms & conditions
facebook

twitter

linkedin

google

youtube

vimeo
[26/06, 8:49 am] +92 324 2921513: Skip to main content
Toggle navigation

Search StratOG
HomeExam preparationPart 1 MRCOGSBAs for Part 1 MRCOG online
resourceAnatomy page 1
Tutorial history
Progress 100% complete
Submit
Anatomy page 1
Assessment History
Total Attempts: 5
Highest Score: 100 %
View last results
View highest result
Retake quiz
Your result100%
The median umbilical ligament is the embryological remnant of what fetal
structure?

Your answer:
Urachus
Correct answer:
Urachus
The answer is urachus. The urachus forms the median umbilical ligament. Be
careful not to confuse it with the medial umbilical ligament, which represents
the remnants of fetal umbilical arteries. Note there is an umbilical vein, not
veins, and this forms the ligamentum teres hepatis.

The superficial inguinal ring is strengthened posteriorly by what structure?

Your answer:
Falx inguinalis
Correct answer:
Falx inguinalis
The answer is the falx inguinalis. The falx inguinalis is also called the conjoint
tendon and strengthens this anatomical area of weakness of the abdominal
wall. Note that although the superficial inguinal tendon sounds plausible, it
actually does not exist.

The inguinal canal transmits the round ligament and which important nerve?

Your answer:
Ilioinguinal nerve
Correct answer:
Ilioinguinal nerve
The answer is the ilioinguinal nerve. It orginates from the anterior rami of L1
and supplies the skin on the mons pubis and labia majora. It also has motor
supply to the internal oblique and transversus abdominis so it strengthens the
conjoint tendon.

A urinary catheter is vigorously inflated in the urethra at the time of caesarean


section. Urine leaks from the rupture into which anatomical space?

Your answer:
Into the anterior abdominal wall and mons pubis
Correct answer:
Into the anterior abdominal wall and mons pubis
The answer is into the anterior abdominal wall and mons pubis. This is the only
route possible due to the anatomy of the urethra.
Which group of nerves all constitute branches of the pudendal nerve?

Your answer:
Perineal nerve, dorsal clitoral nerve, inferior rectal nerve
Correct answer:
Perineal nerve, dorsal clitoral nerve, inferior rectal nerve
The answer is the perineal nerve, dorsal clitoral nerve and inferior rectal nerve.
These are the three important branches of the pudendal nerve.

The inferior hypogastric plexus is an important plexus supplying the hind gut.
What are its nerve root origins?

Your answer:
T10–12 and L1–L2
Correct answer:
T10–12 and L1–L2
The answer is T10–12 and L1–L2. The inferior hypogastric plexus is a
continuation of both hypogastric nerves. It is derived from the lower three
thoracic and upper two lumbar segments. The parasympathetic and
sympathetic supply of this plexus effects uterine contractions and
vasoconstriction.

What three muscles form the superficial group of muscles of the perineum?

Your answer:
Bulbospongiosus, superficial transverse perineal muscle, and ischiocavernous
Correct answer:
Bulbospongiosus, superficial transverse perineal muscle, and ischiocavernous
The answer is bulbospongiosus, superficial transverse perineal muscle and
ischiocavernous. These three muscles make up the superficial group of
perineal muscles. The bulbospongiosus is superficial to the Bartholin's glands.

Sympathetic supply to the bladder is derived from which lumbar segments?

Your answer:
T10 and L2
Correct answer:
T10 and L2
The answer is T10 and L2. Nerve root supply for pelvic organs is a common
single best answer examination question. It is worth drawing a table for each
of the main pelvic organs and writing down the artery, venous, lymphatic and
nerve supply. Note that the parasympathetic supply for the bladder comes
from S2, S3 and S4 but the sympathetic is as above. Sensory nerves are found
in both supplies.

The lower third of the vagina has lymphatic drainage to which lymphatic
nodes?

Your answer:
Superficial inguinal nodes
Correct answer:
Superficial inguinal nodes
The answer is the superficial inguinal nodes. Remember the embryological
difference between the lower one-third of the vagina and the
[26/06, 8:49 am] +92 324 2921513: Skip to main content

Toggle navigation

Search StratOG

Home

Exam preparation

Part 1 MRCOG

SBAs for Part 1 MRCOG online resource

Anatomy page 2

Tutorial history

Progress

100% complete

Submit

Anatomy page 2
Assessment History

Total Attempts:6

Highest Score:100 %

View last resultsView highest resultRetake quiz

Your result60%

What artery supplies the distal third of the transverse colon?

Your answer:

Inferior mesenteric artery

Correct answer:

Inferior mesenteric artery

The correct answer is the inferior mesenteric artery.

What nerve supplies to the anterior one-third of the labium majus?


Your answer:

Pudendal nerve

View correct answer

Correct answer:

Ilioinguinal nerve

The correct answer is ilioinguinal nerve. Posterior two-thirds of the perineal


branch of the pudendal nerve.

What part of the spermatozoa has the highest concentration of mitochondria?

Your answer:

Acrosome

View correct answer

Correct answer:

Middle piece
The correct answer is the middle piece.

Which testicular cells are located outside the blood/testis barrier?

Your answer:

Leydig

Correct answer:

Leydig

The correct answer is leydig cells.

What type of cells are sertoli cells as defined by shape?

Your answer:

Epithelia with gap junctions

View correct answer


Correct answer:

Tall columnar

The correct answer is tall columnar.

What is the layer between the theca cells and the mural granulosa?

Your answer:

Zona pellucida

View correct answer

Correct answer:

Basal lamina

The correct answer is basal lamina.

With regard to ovulation, what is the fluid filled space in a follicle called?
Your answer:

Antrum

Correct answer:

Antrum

The correct answer is antrum.

What size is a resting follicle?

Your answer:

10 mm

View correct answer

Correct answer:

0.02 mm

The correct answer is 0.02 mm.


What size is a follicle that is ready to ovulate?

Your answer:

20 mm

Correct answer:

20 mm

The correct answer is 20 mm.

Regarding the urinary bladder, which of the following is correct?

Your answer:

The bladder wall consists of two layers of detrusor muscle

View correct answer

Correct answer:
A glycosaminoglycan layer coats the luminal surface of the bladder mucosa

The correct answer is a glycosaminoglycan layer coats the luminal surface of


the bladder mucosa. The bladder mucosa is transitional epithelium and is
loosely connected to the muscular bladder wall by the lamina propria that
serves as a connective-tissue layer. The bladder submucosa or lamina propria is
rich in microvasculature and overlies the detrusor muscle. At the trigone, the
epithelium is more densely adherent to the underlying muscle. A layer of fatty
connective tissue covers most of the anterior and lateral bladder in the
retropubic space and can be viewed cystoscopically in the case of a bladder
perforation. The peritoneum covers the bladder posteriorly and separates it
from the anterior cul-de-sac or vesicouterine pouch and abdominal cavity
contents.

The internal pudendal artery leaves the pelvis via what structure?

Your answer:

Greater sciatic foramen

Correct answer:

Greater sciatic foramen

The correct answer is the greater sciatic foramen. The internal pudendal artery
is an artery that branches off the internal iliac artery, providing blood to the
external genitalia. The internal pudendal artery is the terminal branch of the
anterior trunk of the internal iliac artery. It is smaller in the female than in the
male. It exits the pelvic cavity through the greater sciatic foramen, inferior to
the piriformis muscle, to enter the gluteal region. It then curves around the
sacrospinous ligament to enter the perineum through the lesser sciatic
foramen. It travels through the pudendal canal with the internal pudendal
veins and the pudendal nerve.

A direct inguinal hernia that develops from the weakness of the conjoint
tendon can cause damage to what nerve?

Your answer:

Ilioinguinal nerve

Correct answer:

Ilioinguinal nerve

The correct answer is ilioinguinal nerve.

Lymphatic drainage of the uterus is primarily to:

Your answer:
Internal iliac nodes

Correct answer:

Internal iliac nodes

The correct answer is internal iliac nodes.

From which embryological structure does the round ligament of the uterus
originate?

Your answer:

Gubernaculum

Correct answer:

Gubernaculum

The correct answer is gubernaculum.

Which of the following is NOT a branch of the anterior division of the internal
iliac artery?
Your answer:

Ilieolumbar artery

Correct answer:

Ilieolumbar artery

The correct answer is the ilieolumbar artery.

Add/Edit reflective notes

< Previous pageNext page >

Royal College of Obstetricians and Gynaecologists

© 2018

Registered charity no. 213280

27 Sussex Place
Regent's Park
London NW1 4RG
UK

Tel +44 20 7772 6200

Fax +44 20 7723 0575

Contact Us

Accessibility

Privacy & cookie policy

Terms & conditions

facebook

twitter

linkedin
google

youtube

vimeo


[26/06, 8:49 am] +92 324 2921513: Skip to main content
Toggle navigation

Search StratOG
HomeExam preparationPart 1 MRCOGSBAs for Part 1 MRCOG online
resourceBiochemistry page 1
Tutorial history
Progress 100% complete
Submit
Biochemistry page 1
Assessment History
Total Attempts: 7
Highest Score: 100 %
View last results
View highest result
Retake quiz
Your result89%
Which of the following amino acid bases is not contained in RNA?

Your answer:
Thymine
Correct answer:
Thymine
The answer is Thymine. In RNA the thymine is replaced by uracil. The other
bases (adenine, cystosine and guanine) remain the same.

Which of the following blood clotting factors activates fibrinogen in the


common pathway?

Your answer:
Prothrombin
Correct answer:
Prothrombin
The answer is Prothrombin. Accelerin (V) stimulates activation of prothrombin
(II) to thrombin (II), which activates fibrinogen (I) to form fibrin clot. This is
stabilised by cross linking with the aid of fibrin-stabilising factor (XIII).
Remember heparin increases the rate of complex formation of antithrombin
III, which inactivates thrombin.

Which part of the cell cycle is noted for sister chromatids separating and
moving to opposite sides of the cell?

Your answer:
Anaphase
Correct answer:
Anaphase
The answer is Anaphase. Within the cell cycle DNA replication occurs in the
synthesis phase before entering the Gap 2 phase. Mitosis starts with
chromosomes condensing (prophase) follow by chromatids aligning
(metaphase), following by sister chromatids separating and moving
(anaphase). Finally the cell membrane divides in telophase.

Which biochemical technique is used to detect the presence and the amount
of a protein?

Your answer:
Western blotting
Correct answer:
Western blotting
The answer is Western blotting. Southern blotting (named after Edwin
Southern) is used to detect particular sequences to DNA. Northern blotting is
used for the same reason but for RNA. Polymerase chain reaction is used to
amplify very small amounts of DNA (although it can be used for RNA if
converted to DNA). X-ray crystallography is used to reveal the structure of
nucleic acids such as DNA and RNA.

What molecules generate weak forces that can attract neutrophils to cell
walls?

Your answer:
Selectins
Correct answer:
Selectins
The answer is selectins. Selectins are molecules that are expressed in
endometrial cell walls and also leucocytes. They generate weak attraction
forces. Inflammatory processes via interleukins increase the amount of
selectins present and hence, attract more leucocytes.

Ribosomes are located in which part of the cell?

Your answer:
Rough endoplasmic reticulum
Correct answer:
Rough endoplasmic reticulum
The answer is Rough endoplasmic reticulum. Ribosomes 'read' the messenger
RNA created from the nuclear DNA and translate it to create proteins via
transfer RNA. There are free ribosomes in the cytoplasm of cells but they are in
abundance in the rough endoplasmic reticulum.

Which ketone bodies can be used as an energy supply for the brain in
starvation?

Your answer:
Acetoacetate
Correct answer:
Acetoacetate
The answer is acetoacetate. Acetone is also a ketone body but is not used for
energy. It is excreted in urine and is also exhaled in the breath.

You have been asked to review an asymptomatic patient in the early


pregnancy unit. Ultrasound scan is negative for pregnancy. βhCG levels over 48
hours are 550 and 350, and serum progesterone levels are 17 nmol. What is
the most likely outcome?
Your answer:
Non-viable pregnancy with the possibility of spontaneous resolution
Correct answer:
Non-viable pregnancy with the possibility of spontaneous resolution
The correct answer is non-viable pregnancy with the possibility of spontaneous
resolution. Serum progesterone levels are useful adjuncts in the diagnosis of
pregnancy of unknown location. Levels below 15-25 are indicative of non-
viable pregnancy. Results below 20 nmol/l are associated with spontaneous
resolution with sensitivity of 93% and specificity of 94%. Levels above 60
strongly indicate development into normal pregnancy in the future. Serum hCG
levels are us
[26/06, 8:49 am] +92 324 2921513: Skip to main content
Toggle navigation

Search StratOG
HomeExam preparationPart 1 MRCOGSBAs for Part 1 MRCOG online
resourceBiochemistry page 1
Tutorial history
Progress 100% complete
Submit
Biochemistry page 1
Assessment History
Total Attempts: 7
Highest Score: 100 %
View last results
View highest result
Retake quiz
Which of the following amino acid bases is not contained in RNA?

Adenine
Cytosine
Guanine
Thymine
Uracil
Which of the following blood clotting factors activates fibrinogen in the
common pathway?

Factor VIII
Factor XIII
Kallikrein
Kinin
Prothrombin
Which part of the cell cycle is noted for sister chromatids separating and
moving to opposite sides of the cell?

Anaphase
Metaphase
Prophase
Synthesis phase
Telophase
Which biochemical technique is used to detect the presence and the amount
of a protein?

Northern blotting
Polymerase chain reaction
Southern blotting
Western blotting
X-ray crystalography
What molecules generate weak forces that can attract neutrophils to cell
walls?

Cadhedins
Eicosanoids
Hemidesmosomes
Integrins
Selectins
Ribosomes are located in which part of the cell?

Golgi complex
Lysosomes
Mitochondria
Rough endoplasmic reticulum
Smooth endoplasmic reticulum
Which ketone bodies can be used as an energy supply for the brain in
starvation?

Acetyl CoA
Acetone
Acetoacetate
Choline
Succinate
You have been asked to review an asymptomatic patient in the early
pregnancy unit. Ultrasound scan is negative for pregnancy. βhCG levels over 48
hours are 550 and 350, and serum progesterone levels are 17 nmol. What is
the most likely outcome?

Ectopic pregnancy
Failing pregnancy
High risk of ectopic pregnancy needing intervention for treatment
Non-viable pregnancy with the possibility of spontaneous resolution
Viable pregnancy
Your consultant has asked you to send a urea and electrolytes test (U&E) for a
postoperative woman who has undergone a prolonged and difficult
transcervical resection of fibroids. What electrolyte disturbance can occur after
this operation?

Hypernatremia
Hypokalaemia
Hyponatraemia
Hypovolaemia
Increased serum osmolality
In a pregnant woman with diabetes mellitus, target levels of HbA1C should be
below what level?

4.5%
6.5%
8.5%
10.5%
12.5%
A 14-year-old child presents to the adolescent gynaecology clinic. She has a
history of virilisation after undergoing pubertal changes. The karyotype reveals
46XY. An ultrasound scan does not show the presence of a uterus and ovaries.
Which enzyme deficiency may be associated with these clinical features?

5-alpha-reductase deficiency

Complete androgen insensitivity syndrome

Kallmann syndrome

Polycystic ovary syndrome

Turner syndrome

Where is a fetus is alpha-fetoprotein synthesised?

Gut
Kidney
Pancreas
Placenta
Spleen
Which body fat has a major role in gene transcription?

Eicosanoids
Fatty acids
Membrane phospholipids
Phospholipids
Triglycerides
What electrolyte has a role in maintaining intracellular fluids, helping the
regulation of pH and establishing resting membrane potential of the cells?

Bicarbonates
Calcium
Chloride
Potassium
Sodium
Which biochemical process creates pyruvate as an energy supply?

Beta oxidation
Glycolysis
Gluconeogenesis
Oxidative deamination
The citric acid cycle
Chronic renal failure can produce what effect on the parathyroid system?

Hypoparathyroidism
Primary hyperparathyroidism
Pseuohypoparathyroidism
Secondary hyperparathyroidism
Tertiary hyperparathyroidism
An active 70-year-old woman is admitted to the gynaecology clinic for a sling
procedure with a known history of Paget’s disease of the bone. Which
biochemical findings are most likely?

Elevated calcium, elevated phosphate and elevated alkaline phosphatise


Elevated calcium, reduced phosphate and reduced alkaline phosphatise
Normal calcium, normal phosphate and elevated alkaline phosphatase
Reduced calcium, elevated phosphate and elevated alkaline phosphatise
Reduced calcium, reduced phosphate and elevated alkaline phosphatase
Glycated haemoglobin (HbA1C) is a marker of diabetic control over what time
period?

48 hours
Up to 1 week
4–8 w
[26/06, 8:49 am] +92 324 2921513: Skip to main content
Toggle navigation

Search StratOG
HomeExam preparationPart 1 MRCOGSBAs for Part 1 MRCOG online
resourceBiochemistry page 2
Tutorial history
Progress 100% complete
Submit
Biochemistry page 2
Assessment History
Total Attempts: 6
Highest Score: 100 %
View last results
View highest result
Retake quiz
What is the primary source of negatively charged ions in the blood?

Ammonium and chloride


Bicarbonate and ammonium
Bicarbonate and chloride
Phosphate and bicarbonate
Phosphate and chloride
What is the coagulation factor number for fibrinogen?

Factor I
Factor II
Factor IV
Factor V
Factor X
What is the average haemoglobin concentration in a newborn at term?

10 g/dl
12 g/dl
15 g/dl
17 g/dl
25 g/dl
What factor shifts the O2 dissociation curve to the right (i.e., reduces Hb
affinity for 02)?

Decreased CO2 concentration


Decreased hydrogen ion concentration
Increased 2,3 DPG
Increased sodium concentration
Reduced temperature
What coagulation factors are reduced in pregnancy?

III, IV
IX, X
V, VII
XI, XII
XI, XIII
Where is the chloride shift phenomenon seen?

Distal convoluted tubules


Glomerulus
Platelets
Red blood cells
White blood cells
What is the major hydrogen ion buffer in blood?

Albumin
Bicarbonate
Haemoglobin
Phosphate
Uric acid
What is the major production mechanism of 2,3 DPG?

Anaerobic metabolism in smooth muscle


Gluconeogenesis
Glycolysis
Only in the liver
Suprarenal glands
With regard to plasma calcium, what proportion is carried in an ionised form?

1%
5%
45%
75%
78%
With regard to bound plasma calcium, what is the major carrier?

Calciprotein transfer molecule 1 (CTM 1)


Haemoglobin
Neutrophils
Plasma proteins
SHBG
Where in the body is the major production of 1,25 (OH) D3 (calcitriol)?
Bone
Kidneys
Liver
Lung
Parathyroids
With regards to vitamin D metabolism, what food is particularly high in D2?

Bananas
Beef
Oily fish
Soya milk
Wheat
Which of the following best describes the complement system?

It consists of a series of approximately 60 serum glycoproteins


The major function of the complement system is to remove and destroy
antigens
The most abundant complement is C4
The alternative pathway is initiated by binding of antibody to antigen
The classic pathway does not require any antibody
Which coagulation factor decreases in pregnancy?

Factor I
Factor VII
Factor VIII
Factor X
Factor XI
With regard to oral iron absorption, reduction of Fe3+ to Fe2+ is inhibited by
what substance?

Gastric acid
Tea
Vitamin B1
Vitamin C
Vitamin E
A 54-year-old woman presents to her GP with malaise, a history of recent
weight loss and bloating. On examination the GP notes a large pelvic mass. He
sends her for an urgent abdominal ultrasound scan. What tumour marker
would be most useful to aid the diagnosis in this case?

a-FP
CA 15-3
CA 19-9
CA-125
HCG
This is the last question of this quiz. Press finish to submit your choice(s) and
reveal the answer(s).

Finish
Add/Edit reflective notes
< Previous pageNext page >
Royal College of Obstetricians and Gynaecologists
© 2018
Registered charity no. 213280
27 Sussex Place
Regent's Park
London NW1 4RG
UK

Tel +44 20 7772 6200


Fax +44 20 7723 0575
Contact Us
Accessibility
Privacy & cookie policy
Terms & conditions
facebook

twitter

linkedin

google

youtube

vimeo
[26/06, 8:49 am] +92 324 2921513: Skip to main content
Toggle navigation

Search StratOG
HomeExam preparationPart 1 MRCOGSBAs for Part 1 MRCOG online
resourceBiochemistry page 2
Tutorial history
Progress 100% complete
Submit
Biochemistry page 2
Assessment History
Total Attempts: 7
Highest Score: 100 %
View last results
View highest result
Retake quiz
Your result56%
What is the primary source of negatively charged ions in the blood?

Your answer:
Phosphate and bicarbonate
View correct answer
Correct answer:
Bicarbonate and chloride
The correct answer is bicarbonate and chloride.

What is the coagulation factor number for fibrinogen?

Your answer:
Factor I
Correct answer:
Factor I
The correct answer is factor I.

What is the average haemoglobin concentration in a newborn at term?

Your answer:
12 g/dl
View correct answer
Correct answer:
17 g/dl
The correct answer is 17 g/dl.

What factor shifts the O2 dissociation curve to the right (i.e., reduces Hb
affinity for 02)?

Your answer:
Increased 2,3 DPG
Correct answer:
Increased 2,3 DPG
The correct answer is increased 2,3 DPG.
What coagulation factors are reduced in pregnancy?

Your answer:
XI, XIII
Correct answer:
XI, XIII
The correct answers are XI and XIII. the main changes in haemostasis factors
during pregnancy are:

Platelet count ↓
II, V =
Fibrinogen,VII,VIII,von Willebrand factor,IX,X,XII ↑
XI =/↓
XIII ↑/↓
Antithrombin =
Protein C =/↑
Protein S ↓
Heparin cofactor II ↑
F1+2,TAT,d-dimer ↑
t-PA ↓
ELT,PAI, TAFI ↑

Where is the chloride shift phenomenon seen?

Your answer:
Red blood cells
Correct answer:
Red blood cells
The correct answer is red blood cells.

What is the major hydrogen ion buffer in blood?

Your answer:
Haemoglobin
Correct answer:
Haemoglobin
The correct answer is haemoglobin.

What is the major production mechanism of 2,3 DPG?

Your answer:
Only in the liver
View correct answer
Correct answer:
Glycolysis
The correct answer is glycolysis.
With regard to plasma calcium, what proportion is carried in an ionised form?

Your answer:
5%
View correct answer
Correct answer:
45%
The correct answer is 45%.

With regard to bound plasma calcium, what is the major carrier?

Your answer:
Calciprotein transfer molecule 1 (CTM 1)
View correct answer
Correct answer:
Plasma proteins
The correct answer is plasma proteins.

Where in the body is the major production of 1,25 (OH) D3 (calcitriol)?


Your answer:
Kidneys
Correct answer:
Kidneys
The correct answer is the kidneys, specifically the proximal tubes of the
nephrons.

With regards to vitamin D metabolism, what food is particularly high in D2?

Your answer:
Soya milk
View correct answer
Correct answer:
Oily fish
The correct answer is oily fish.

Which of the following best describes the complement system?

Your answer:
The major function of the complement system is to remove and destroy
antigens
Correct answer:
The major function of the complement system is to remove and destroy
antigens
The correct answer is the major function of the complement system is to
remove and destroy antigens.

Which coagulation factor decreases in pregnancy?

Your answer:
Factor XI
Correct answer:
Factor XI
The correct answer is Factor XI.

With regard to oral iron absorption, reduction of Fe3+ to Fe2+ is inhibited by


what substance?

Your answer:
Gastric acid
View correct answer
Correct answer:
Tea
The correct answer is tea.
A 54-year-old woman presents to her GP with malaise, a history of recent
weight loss and bloating. On examination the GP notes a large pelvic mass. He
sends her for an urgent abdominal ultrasound scan. What tumour marker
would be most useful to aid the diagnosis in this case?

Your answer:
CA-125
Correct answer:
CA-125
The best answer is CA-125.

Add/Edit reflective notes


< Previous pageNext page >
Royal College of Obstetricians and Gynaecologists
© 2018
Registered charity no. 213280
27 Sussex Place
Regent's Park
London NW1 4RG
UK

Tel +44 20 7772 6200


Fax +44 20 7723 0575
Contact Us
Accessibility
Privacy & cookie policy
Terms & conditions
fac
[26/06, 8:49 am] +92 324 2921513: Skip to main content
Toggle navigation

Search StratOG
HomeExam preparationPart 1 MRCOGSBAs for Part 1 MRCOG online
resourceBiochemistry page 3
Tutorial history
Progress 100% complete
Submit
Biochemistry page 3
Assessment History
Total Attempts: 5
Highest Score: 100 %
View last results
View highest result
Retake quiz
What is the action of cytochrome P450 enzymes?

Always acts an isomerase


Always acts as a transferase
Always adds an alcohol group
Always adds a terminal electron
Always catalyses hydroxylations
Which cells in the body are dependent upon anaerobic respiration?

Cardiac muscle
Hepatocytes
Nephrons
Red blood cells
White blood cells
What is the product of aerobic glycolysis?

Free fatty acids


Glycogen
Lactate
Pyruvate
Urea
Where in the body is ATP found?

Both intracellular and extracellular


Only extracellular
Only intracellular
Only within high energy output cells
Only within mitochondria
With regard to Conns syndrome, what are the likely changes in pH, potassium
and sodium concentration?

Hypernatraemia, hyperkalaemia, alkalosis


Hypernatraemia, hypokalaemia, acidosis
Hypernatraemia, hypokalaemia, alkalosis
Hyponatraemia, hyperkalaemia acidosis
Hyponatraemia, hypokalaemia, acidosis
What type of acid base disturbance is indicated by the following?

pH: 7.2 (7.35–7.45)


PaCO2: 4.2 kPa (4.5–6.0)
BE: -18 (–2 to +2).
Bicarbonate renal excess
Metabolic acidosis
Metabolic alkalosis
Respiratory acidosis
Respiratory alkalosis
What is the most important buffer in the interstitial space?

Ammonia
Bicarbonate
Chloride
Phosphate
Plasma proteins
Ammonia is made from what amino acid in the kidney?

Arginine
Glutamine
Histidine
Lysine
Serine
What is the main buffer in urine?

Ammonia
Bicarbonate
Chloride
Haemoglobin
Phosphate
Bicarbonate is reabsorbed in what part of the kidney?

Collecting ducts
Distal convoluted tubules
Glomerulus
Loop of Henle
Proximal convoluted tubules
With regard to transport in the red blood cells, what ion enters the cell to
maintain electrical neutrality with the outward movement of bicarbonate?

Ammonia
Ammonium
Chloride
Potassium
Sodium
This is the last question of this quiz. Press finish to submit your choice(s) and
reveal the answer(s).

Finish
Add/Edit reflective notes
< Previous pageNext page >
Royal College of Obstetricians and Gynaecologists
© 2018
Registered charity no. 213280
27 Sussex Place
Regent's Park
London NW1 4RG
UK

Tel +44 20 7772 6200


Fax +44 20 7723 0575
Contact Us
Accessibility
Privacy & cookie policy
Terms & conditions
facebook

twitter

linkedin

google

youtube
vimeo
[26/06, 8:49 am] +92 324 2921513: Skip to main content
Toggle navigation

Search StratOG
HomeExam preparationPart 1 MRCOGSBAs for Part 1 MRCOG online
resourceBiochemistry page 3
Tutorial history
Progress 100% complete
Submit
Biochemistry page 3
Assessment History
Total Attempts: 6
Highest Score: 100 %
View last results
View highest result
Retake quiz
Your result73%
What is the action of cytochrome P450 enzymes?

Your answer:
Always catalyses hydroxylations
Correct answer:
Always catalyses hydroxylations
The correct answer is always catalyse hydroxylations.
Which cells in the body are dependent upon anaerobic respiration?

Your answer:
Cardiac muscle
View correct answer
Correct answer:
Red blood cells
The correct answer is red blood cells.

What is the product of aerobic glycolysis?

Your answer:
Pyruvate
Correct answer:
Pyruvate
The correct answer is pyruvate.

Where in the body is ATP found?

Your answer:
Only intracellular
Correct answer:
Only intracellular
The correct answer is only intracellular.

With regard to Conns syndrome, what are the likely changes in pH, potassium
and sodium concentration?

Your answer:
Hypernatraemia, hypokalaemia, alkalosis
Correct answer:
Hypernatraemia, hypokalaemia, alkalosis
The correct answer is hypernatraemia, hypokalaemia, alkalosis.

What type of acid base disturbance is indicated by the following?

pH: 7.2 (7.35–7.45)


PaCO2: 4.2 kPa (4.5–6.0)
BE: -18 (–2 to +2).
Your answer:
Metabolic acidosis
Correct answer:
Metabolic acidosis
The correct answer is partially compensated metabolic acidosis.
What is the most important buffer in the interstitial space?

Your answer:
Plasma proteins
View correct answer
Correct answer:
Bicarbonate
The correct answer is bicarbonate.

Ammonia is made from what amino acid in the kidney?

Your answer:
Glutamine
Correct answer:
Glutamine
The correct answer is glutamine. Remember glutaminase!

What is the main buffer in urine?

Your answer:
Phosphate
Correct answer:
Phosphate
The correct answer is phosphate.

Bicarbonate is reabsorbed in what part of the kidney?

Your answer:
Proximal convoluted tubules
Correct answer:
Proximal convoluted tubules
The correct answer is the proximal convoluted tubules.

With regard to transport in the red blood cells, what ion enters the cell to
maintain electrical neutrality with the outward movement of bicarbonate?

Your answer:
Sodium
View correct answer
Correct answer:
Chloride
The correct answer is chloride.

Add/Edit reflective notes


< Previous pageNext page >
Royal College of Obstetricians and Gynaecologists
© 2018
Registered charity no. 213280
27 Sussex Place
Regent's Park
London NW1 4RG
UK

Tel +44 20 7772 6200


Fax +44 20 7723 0575
Contact Us
Accessibility
Privacy & cookie policy
Terms & conditions
facebook

twitter

linkedin

google

youtube

vimeo
[26/06, 8:49 am] +92 324 2921513: Skip to main content
Toggle navigation
Search StratOG
HomeExam preparationPart 1 MRCOGSBAs for Part 1 MRCOG online
resourceEmbryology
Tutorial history
Progress 100% complete
Submit
Embryology
Assessment History
Total Attempts: 6
Highest Score: 100 %
View last results
View highest result
Retake quiz
Your result89%
How many oocytes are available at the time of puberty?

Your answer:
400 000
Correct answer:
400 000
The answer is 400 000. There are 1-2 million primordial follicles present at
birth but through the atretic process only 400 000 oocytes are available at
puberty.

A cystic structure is noted lateral to the vagina. This is believed to be a


Gartners duct. What is the embryological origin of this structure?
Your answer:
Mesonephric duct
Correct answer:
Mesonephric duct
The answer is Mesonephric duct. The mesonephric duct forms the ureter and
collecting system and in the female can form the epoophoron, appendix
vesiculosa and Gartner's duct.

Which sperm cell type undergoes the second meiotic division?

Your answer:
Secondary spermatocytes
Correct answer:
Secondary spermatocytes
The answer is Secondary spermatocytes. Spermatogoniun are diploid and
divide by mitosis to produce diploid primary spermatocytes. These divide by
the first stage of meiosis to create haploid secondary spermatocytes.
Secondary spermatocytes complete the second stage of meiosis to become
spermatids. The conversion from spermatids to sperm is known as
spermiogenesis.

The intraembryonic coelom forms within which structure?

Your answer:
Lateral plate mesoderm
Correct answer:
Lateral plate mesoderm
The answer is Lateral plate mesoderm. In the second week of development the
lateral plate mesoderm splits to create this cavity. It will form the space for the
thoracic and abdominal cavity.

What is the best indicator of gestational age on a dating scan in the first
trimester?

Your answer:
Crown rump length
Correct answer:
Crown rump length
The answer is Crown rump length. Biparietal diameter is appropriate after 14
weeks. Gestational sac measurements help in early pregnancy diagnosis.

A 40-year-old woman is noted to have an elevation of alphafetoprotein. Which


is the most likely pathology diagnosed by ultrasound scan?

Your answer:
Spina bifida
Correct answer:
Spina bifida
The answer is Spina bifida. Alphafetoprotein is a glycoprotein synthesised by
three fetal structures: the gut, liver and yolk sac. It is may be elevated in a
defect of the anterior abdominal wall and the neural tube. As these defects are
not covered by skin, AFP freely enters the amniotic fluid from the fetal
circulation, leading to elevation of levels.

The canal of Nuck is associated with what?


Your answer:
Round ligament
Correct answer:
Round ligament
The answer is round ligament. The round ligament arises from the body of the
uterus antero inferior to the cornue. It passes through the layers of broad
ligaments across the psoas and external iliac vessels. It then passes through the
deep inguinal ring and the inguinal canal to the labium majus. In the fetus, the
round ligament is surrounded by a peritoneum, processus vaginalis, which is
obliterated at birth but may remain patent as the canal of Nuck.

In the development of the oocyte, when does the second meiotic division
occur?

Your answer:
At fertilisation
Correct answer:
At fertilisation
The answer is at fertilisation. The primary oocyte resumes meiotic maturation
in response to a surge in luteinising hormone. This results in a polar body and
the secondary oocyte. The oocyte is then ovulated and enters the second
meiotic cycle. It is arrested in metaphase II and only completes meiosis after
the entry of the spermatozoa at fertilisation.

Implantation of the embryo occurs at which stage?

Your answer:
Blastocyst
Correct answer:
Blastocyst
The answer is Blastocyst. Within 24 hours of fertilisation, the zygote undergoes
cleavage. It then further subdivides into blastomeres. By the third day, the
embryo contains 12 cells. By the fourth day, it is comprised of 16-32 cells and is
called a morula. On day 5, the blastocyst hat
[26/06, 8:49 am] +92 324 2921513: Skip to main content

Toggle navigation

Search StratOG

Home

Exam preparation

Part 1 MRCOG

SBAs for Part 1 MRCOG online resource

Endocrinology page 1

Tutorial history

Progress
100% complete

Submit

Endocrinology page 1

Assessment History

Total Attempts:5

Highest Score:100 %

View last resultsView highest resultRetake quiz

What inhibits glucagon releases?

Adrenaline

Exercise

Hypoglycaemia

Increased amino acids


Increased free fatty acids

What is the ratio of testosterone bound to sex hormone-binding globulin


(SHBG) and albumin respectively?

1% free, 19% albumin-bound and 80% SHBG-bound

5% free, 30% albumin-bound and 55% SHBG-bound

5% free, 1% albumin-bound and 94% SHBG-bound

10% free, 30% albumin-bound and 60% SHBG-bound

30% free, 19% albumin-bound and 51% SHBG-bound

What is the most common cause of hyperprolactinaemia?

Macroprolactinoma

Microprolactinoma

Primary hypothyroidism

Renal failure

Pituitary stalk interruption syndrome (PSIS)


Which part of the ovary is responsible for androgen production?

Granulosa cells

Leydig cells

Organ of Zuckerkandl cells

Sertoli cells

Theca cells

What is an appropriate endocrine response to low, detected serum calcium


levels?

Decreased parathyroid syndrome (PTH), increased phosphate excretion,


increased calcitriol and increased calcitonin

Decreased PTH, reduced phosphate excretion, increased calcitriol and


increased calcitonin

Increased PTH, increased phosphate excretion, increased calcitriol and reduced


calcitonin

Increased PTH phosphate excretion, reduced calcitriol and reduced calcitonin

Increased PTH, reduced phosphate excretion, increased calcitriol and reduced


calcitonin
Where in the body is aldosterone synthesized?

Distal tubules

Lung

Zona fasciculata

Zona reticularis

Zone glomerulosa

With regard to the renin-angiotensin system, where are the juxtaglomerular


cells located?

Afferent arteriole in the kidney

Afferent arteriole in the liver

Afferent arteriole in the lung

Efferent arteriole in the kidney

Efferent arteriole in the lung

What are the common features of Cushing syndrome?


Exophthalmos, renal calculi

Headache and proximal myopathy

Moon face and weight gain

Poor wound healing and polycythaemia

Renal calculi and changes in mental health

Which hormone has a structure similar to growth hormone?

Anti-müllerian hormone

Cortisol

Follicle-stimulating hormone

Insulin

Prolactin

Which of the following is the most reliable test for the diagnosis of Cushing
syndrome?

24-hour urine sample detecting cortisol level


Cortisol measurement taken at 0600 hours

Dexamethasone suppression test

Random cortisol measurement

Short adrenocorticotrophic hormone suppression test

What are the classical blood test results in Addison’s disease?

Hyperkalaemia, hyponatraemia, uraemia, hypercalcaemia and basophila

Hyperkalaemia, hyponatraemia, uraemia, hypercalcaemia and eosinophila

Hyperkalaemia, hyponatraemia, uraemia, hypocalcaemia and basophila

Hyperkalaemia, hyponatraemia, uraemia, hypocalcaemia and eosinophila

Hypokalaemia, hypernatraemia, uraemia, hypercalcaemia and eosinophila

Which hypothalamic hormone stimulates the release of prolactin?

Dopamine

Growth hormone releasing hormone


Gonadotrophin-releasing hormone

Somatostatin

Thyrotrophin-releasing hormone

Adrenomedullary hypersecretion is caused by which condition?

Addison’s disease

Conn syndrome

Cushing syndrome

Diabetes insipidus

Phaeochromocytoma

Leptin is a protein hormone secreted by adipose tissue. What stimulates its


release?

Catecholamines

High body mass index

Hypoinsulinaemia
Mineralocorticoids

Stress

Which hormone is responsible for the ovulation and initiation of luteinisation


of the follicle?

Estrogen

Follicle-stimulating hormone

Gonadotrophin-releasing hormone

Luteinising hormone

Progesterone

The blood test of a 36-year-old fertility patient shows elevated luteinising


hormone (LH) levels and high estradiol levels. What is the most likely cause of
this?

Hypogonadotrophic hypogonadism

Midcycle LH surge

Ovarian failure
Polycystic ovary syndrome

Weight-related amenorrhoea

Which hormone is made in the ovarian granulosa cells before ovulation


occurs?

Aldosterone

Alphafetoprotein

Antimüllerian hormone

Progesterone

Testosterone

This is the last question of this quiz. Press finish to submit your choice(s) and
reveal the answer(s).

Finish

Add/Edit reflective notes

< Previous pageNext page >


Royal College of Obstetricians and Gynaecologists

© 2018

Registered charity no. 213280

27 Sussex Place
Regent's Park
London NW1 4RG
UK

Tel +44 20 7772 6200

Fax +44 20 7723 0575

Contact Us

Accessibility

Privacy & cookie policy

Terms & conditions

facebook
twitter

linkedin

google

youtube

vimeo

Skip to main content

Toggle navigation


Search StratOG

Home

Exam preparation

Part 1 MRCOG

SBAs for Part 1 MRCOG online resource

Endocrinology page 1

Tutorial history

Progress

100% complete

Submit

Endocrinology page 1

Assessment History
Total Attempts:5

Highest Score:100 %

View last resultsView highest resultRetake quiz

What inhibits glucagon releases?

Adrenaline

Exercise

Hypoglycaemia

Increased amino acids

Increased free fatty acids

What is the ratio of testosterone bound to sex hormone-binding globulin


(SHBG) and albumin respectively?

1% free, 19% albumin-bound and 80% SHBG-bound

5% free, 30% albumin-bound and 55% SHBG-bound

5% free, 1% albumin-bound and 94% SHBG-bound


10% free, 30% albumin-bound and 60% SHBG-bound

30% free, 19% albumin-bound and 51% SHBG-bound

What is the most common cause of hyperprolactinaemia?

Macroprolactinoma

Microprolactinoma

Primary hypothyroidism

Renal failure

Pituitary stalk interruption syndrome (PSIS)

Which part of the ovary is responsible for androgen production?

Granulosa cells

Leydig cells

Organ of Zuckerkandl cells

Sertoli cells
Theca cells

What is an appropriate endocrine response to low, detected serum calcium


levels?

Decreased parathyroid syndrome (PTH), increased phosphate excretion,


increased calcitriol and increased calcitonin

Decreased PTH, reduced phosphate excretion, increased calcitriol and


increased calcitonin

Increased PTH, increased phosphate excretion, increased calcitriol and reduced


calcitonin

Increased PTH phosphate excretion, reduced calcitriol and reduced calcitonin

Increased PTH, reduced phosphate excretion, increased calcitriol and reduced


calcitonin

Where in the body is aldosterone synthesized?

Distal tubules

Lung

Zona fasciculata
Zona reticularis

Zone glomerulosa

With regard to the renin-angiotensin system, where are the juxtaglomerular


cells located?

Afferent arteriole in the kidney

Afferent arteriole in the liver

Afferent arteriole in the lung

Efferent arteriole in the kidney

Efferent arteriole in the lung

What are the common features of Cushing syndrome?

Exophthalmos, renal calculi

Headache and proximal myopathy

Moon face and weight gain

Poor wound healing and polycythaemia


Renal calculi and changes in mental health

Which hormone has a structure similar to growth hormone?

Anti-müllerian hormone

Cortisol

Follicle-stimulating hormone

Insulin

Prolactin

Which of the following is the most reliable test for the diagnosis of Cushing
syndrome?

24-hour urine sample detecting cortisol level

Cortisol measurement taken at 0600 hours

Dexamethasone suppression test

Random cortisol measurement

Short adrenocorticotrophic hormone suppression test


What are the classical blood test results in Addison’s disease?

Hyperkalaemia, hyponatraemia, uraemia, hypercalcaemia and basophila

Hyperkalaemia, hyponatraemia, uraemia, hypercalcaemia and eosinophila

Hyperkalaemia, hyponatraemia, uraemia, hypocalcaemia and basophila

Hyperkalaemia, hyponatraemia, uraemia, hypocalcaemia and eosinophila

Hypokalaemia, hypernatraemia, uraemia, hypercalcaemia and eosinophila

Which hypothalamic hormone stimulates the release of prolactin?

Dopamine

Growth hormone releasing hormone

Gonadotrophin-releasing hormone

Somatostatin

Thyrotrophin-releasing hormone

Adrenomedullary hypersecretion is caused by which condition?


Addison’s disease

Conn syndrome

Cushing syndrome

Diabetes insipidus

Phaeochromocytoma

Leptin is a protein hormone secreted by adipose tissue. What stimulates its


release?

Catecholamines

High body mass index

Hypoinsulinaemia

Mineralocorticoids

Stress

Which hormone is responsible for the ovulation and initiation of luteinisation


of the follicle?

Estrogen
Follicle-stimulating hormone

Gonadotrophin-releasing hormone

Luteinising hormone

Progesterone

The blood test of a 36-year-old fertility patient shows elevated luteinising


hormone (LH) levels and high estradiol levels. What is the most likely cause of
this?

Hypogonadotrophic hypogonadism

Midcycle LH surge

Ovarian failure

Polycystic ovary syndrome

Weight-related amenorrhoea

Which hormone is made in the ovarian granulosa cells before ovulation


occurs?

Aldosterone
Alphafetoprotein

Antimüllerian hormone

Progesterone

Testosterone

This is the last question of this quiz. Press finish to submit your choice(s) and
reveal the answer(s).

Finish

Add/Edit reflective notes

< Previous pageNext page >

Royal College of Obstetricians and Gynaecologists

© 2018

Registered charity no. 213280

27 Sussex Place
Regent's Park
London NW1 4RG
UK

Tel +44 20 7772 6200

Fax +44 20 7723 0575

Contact Us

Accessibility

Privacy & cookie policy

Terms & conditions

facebook

twitter

linkedin
google

youtube

vimeo


[26/06, 8:49 am] +92 324 2921513: Skip to main content

Toggle navigation

Search StratOG

Home

Exam preparation

Part 1 MRCOG

SBAs for Part 1 MRCOG online resource


Endocrinology page 2

Tutorial history

Progress

100% complete

Submit

Endocrinology page 2

Assessment History

Total Attempts:5

Highest Score:100 %

View last resultsView highest resultRetake quiz

What enzyme converts androstenedione to estradiol?

5-alpha reductase
Catalyse

CYP19 aromatase

Transcriptase

What is the primary treatment of hyperthyroidism in pregnancy?

Carbimazole

Labetalol

Propylthiouracil

Radioactive iodine

Surgery

What is the most common cause of hyperthyroidism in the non-pregnant


woman?

Adenoma

Graves disease

Iodine excess
Subacute thyroiditis

Toxic multinodular goitre

What is the definition of premature menopause?

Menopause before 30 years of age

Menopause before 35 years of age

Menopause before 40 years of age

Menopause before 45 years of age

Menopause before 50 years of age

What is the most common cause of delayed puberty?

Constitutional delay

Low BMI

Ovarian failure

Pituitary failure
Turner syndrome

What is the definition of puberty in women?

Demonstration of stage 4 breast development

Demonstration of stage 5 breast development

Maturation of the HPO axis where by FSH secretion is greater than 10 IU

Physical maturation whereby the women becomes capable of sexual


reproduction

Physical maturation whereby the women first menstruates

Which three hormones consist of almost identical alpha chains?

ADH, oxytocin, FSH

FSH, LH, growth hormone

HCG, FSH, LH

HCG, ADH and FSH

TSH, growth hormone, HCG


What hormone is secreted by acidophil cells in the anterior pituitary?

Oxytocin

Growth hormone

FSH

LH

TSH

What hormone is significantly increased on day 0 of the menstrual cycle?

Estradiol

FSH

Inhibin

LH

Progesterone

With regard to the ovary, what cells produce testosterone?


Basal lamina

Cumulus granulosa

Leydig

Mural granulosa

Theca

What is the finite life span of the corpus luteum?

8 days

10 days

12 days

14 days

16 days

How many hours after the LH surge does ovulation occur?

3–6 hours
12–18 hours

24–36 hours

48–72 hours

Where in the body is calcidiol produced?

Kidneys

Liver

Parathyroids

Skin

Spleen

A 16-year-old primiparous woman presents to the Early Pregnancy Unit with a


two-day history of PV bleeding. She is approximately 7 weeks in to a
spontaneous unplanned pregnancy. She is otherwise fit and well. Her serum
βhCG is 94 000 iu/l. During pregnancy, from what structure is βhCG produced?

Cytotrophoblast

Decidua
Wharton’s jelly of umbilical cord

Secretory glands in endometrium

Syncytiotrophoblast

This is the last question of this quiz. Press finish to submit your choice(s) and
reveal the answer(s).

Finish

Add/Edit reflective notes

< Previous pageNext page >

Royal College of Obstetricians and Gynaecologists

© 2018

Registered charity no. 213280

27 Sussex Place
Regent's Park
London NW1 4RG
UK
Tel +44 20 7772 6200

Fax +44 20 7723 0575

Contact Us

Accessibility

Privacy & cookie policy

Terms & conditions

facebook

twitter

linkedin

google
youtube

vimeo


[26/06, 8:49 am] +92 324 2921513: Skip to main content

Toggle navigation

Search StratOG

Home

Exam preparation

Part 1 MRCOG

SBAs for Part 1 MRCOG online resource

Endocrinology page 1
Tutorial history

Progress

100% complete

Submit

Endocrinology page 1

Assessment History

Total Attempts:6

Highest Score:100 %

View last resultsView highest resultRetake quiz

Your result88%

What inhibits glucagon releases?

Your answer:
Increased free fatty acids

Correct answer:

Increased free fatty acids

The answer is increased free fatty acids. Glucagon is secreted by the alpha cells
of the pancreas. It increases hepatic glycogenolysis and also glyconeogenesis.
Adrenaline stimulates glucagon release in times of stress .This stimulates liver
glycogenolysis to ensure maximum glucose output to prepare for ‘flight or
fight' reaction. Hyperglycaemia inhibits glucagon release.

What is the ratio of testosterone bound to sex hormone-binding globulin


(SHBG) and albumin respectively?

Your answer:

1% free, 19% albumin-bound and 80% SHBG-bound

Correct answer:

1% free, 19% albumin-bound and 80% SHBG-bound

The answer is 1% free, 19% albumin-bound and 80% SHBG-bound. Only 1% is


free but it is the albumin-bound fraction that is hormonally active.

What is the most common cause of hyperprolactinaemia?


Your answer:

Primary hypothyroidism

Correct answer:

Primary hypothyroidism

The answer is Primary hypothyroidism. Other common causes include


neuroleptic medication and dopamine D2 receptor anti-emetics.

Which part of the ovary is responsible for androgen production?

Your answer:

Theca cells

Correct answer:

Theca cells

The answer is Theca cells. Luteneising hormone drives the conversion of


cholesterol into androgens in the theca cells. These androgens are then
transferred to the granulosa cells for conversion (aromatisation) into estrogen
(estradiol) under the influence of follicle-stimulating hormones.
What is an appropriate endocrine response to low, detected serum calcium
levels?

Your answer:

Increased PTH, increased phosphate excretion, increased calcitriol and reduced


calcitonin

Correct answer:

Increased PTH, increased phosphate excretion, increased calcitriol and reduced


calcitonin

The answer is increased PTH, increased phosphate excretion, increased


calcitriol and reduced calcitonin. Low calcium levels are detected by the
parathyroid glands and PTH is secreted. PTH acts on the kidneys to increase
calcium absorption and also increase phosphate excretion. PTH also increases
the activity of 25-hydroxycholecalciferol, which converts calcidiol to calcitriol.

Where in the body is aldosterone synthesized?

Your answer:

Zona reticularis

View correct answer

With regard to the renin-angiotensin system, where are the juxtaglomerular


cells located?
Your answer:

Afferent arteriole in the kidney

Correct answer:

Afferent arteriole in the kidney

The answer is afferent arteriole in the kidney. Juxtaglomerular apparatus is


made up of the juxtaglomerular cells, lacis cells and macula densa. Renin is
secreted from the juxtaglomerular cells in response to a decrease in arterial
blood pressure (detected by the baroreceptors), a decrease in sodium chloride
(detected by the macula densa) and the sympathetic nervous system.

What are the common features of Cushing syndrome?

Your answer:

Moon face and weight gain

Correct answer:

Moon face and weight gain

The answer is moon face and weight gain. Although all of the above are
features of Cushing syndrome, only the following are common: moon face,
weight increase and central weight increase, changes in mental health,
impaired glucose tolerance, acne, bruising and proximal myopathy.

Which hormone has a structure similar to growth hormone?

Your answer:

Prolactin

Correct answer:

Prolactin

The answer is Prolactin. Growth hormone is structurally similar to prolactin


and human placental lactogen.

Which of the following is the most reliable test for the diagnosis of Cushing
syndrome?

Your answer:

Dexamethasone suppression test

Correct answer:

Dexamethasone suppression test


The answer is Dexamethasone suppression test. A 24-hour urine sample to
detect cortisol can be used for outpatients; however, a suppression test is
more reliable.

What are the classical blood test results in Addison’s disease?

Your answer:

Hyperkalaemia, hyponatraemia, uraemia, hypercalcaemia and basophila

View correct answer

Which hypothalamic hormone stimulates the release of prolactin?

Your answer:

Thyrotrophin-releasing hormone

Correct answer:

Thyrotrophin-releasing hormone

The answer is Thyrotropin-releasing hormone. Thyrotrophin-releasing


hormone stimulates the release of TSH and prolactin from the anterior
pituitary. Dopamine also released from the hypothalamus inhibits prolactin
release.

Adrenomedullary hypersecretion is caused by which condition?


Your answer:

Phaeochromocytoma

Correct answer:

Phaeochromocytoma

The answer is Phaeochromocytoma. Adrenomedullary hypersecretion is


caused by catecholamine-secreting tumours. It is characterised by
hypertension, pallor, headache and sweating and glucose intolerance. Conn
syndrome is associated with hypersecretion of aldosterone and is caused by an
adrenal adenoma. Diabetes insipidus is associated with the abscence of
antidiuretic hormone.

Leptin is a protein hormone secreted by adipose tissue. What stimulates its


release?

Your answer:

High body mass index

Correct answer:

High body mass index


The answer is High body mass index. Leptin is a 167-amino acid product
derived from fat cells. Leptin receptors are found in the ovaries, the uterus, the
heart and skeletal tissue. Leptin provides a signal, reflecting the energy states
of the body, and may be involved in the control of appetite and reproduction.
It is a probable link between weight loss and menstruation. It is stimulated by
glucocorticoids, high body mass index, long-term hyperinsulinaemia and
excessive food ingestion.

Which hormone is responsible for the ovulation and initiation of luteinisation


of the follicle?

Your answer:

Luteinising hormone

Correct answer:

Luteinising hormone

The answer is Luteinising hormone (LH). Circulating estrogen levels rise in the
follicular phase and are responsible for the preovulatory surge of LH. This then
triggers the ovulation and initiates the luteinisation. Progesterone may have
some role in the LH surge, but the main factor is LH, which is responsible for
the above actions.

The blood test of a 36-year-old fertility patient shows elevated luteinising


hormone (LH) levels and high estradiol levels. What is the most likely cause of
this?

Your answer:
Midcycle LH surge

Correct answer:

Midcycle LH surge

The answer is Midcycle LH surge. Polycystic ovary syndrome (PCOS) is


associated with high LH levels and usually normal estradiol levels. Ovarian
failure will result in high LH levels but low estradiol levels. Hypogonadotrophic
hypogonadism and weight-related amenorrhoea are associated with low LH
levels and low estradiol levels. PCOS results in a ratio of LH/Follicle-stimulating
hormone (FSH) > 1. The blood test may have been taken midcycle. In this case
we are not able to diagnose PCOS because we do not have a value for FSH.
Generally, women with PCOS have normal estradiol levels.

Which hormone is made in the ovarian granulosa cells before ovulation


occurs?

Your answer:

Antimüllerian hormone

Correct answer:

Antimüllerian hormone
The answer is Antimüllerian hormone. The granulosa cells also convert
androgens produced by the theca cells into estrogen, and after ovulation, they
produce progesterone in the granulosa lutein cells.

Add/Edit reflective notes

< Previous pageNext page >

Royal College of Obstetricians and Gynaecologists

© 2018

Registered charity no. 213280

27 Sussex Place
Regent's Park
London NW1 4RG
UK

Tel +44 20 7772 6200

Fax +44 20 7723 0575

Contact Us

Accessibility
Privacy & cookie policy

Terms & conditions

facebook

twitter

linkedin

google

youtube

vimeo

[26/06, 8:49 am] +92 324 2921513: Skip to main content

Toggle navigation

Search StratOG

Home

Exam preparation

Part 1 MRCOG

SBAs for Part 1 MRCOG online resource

Endocrinology page 2

Tutorial history

Progress

100% complete
Submit

Endocrinology page 2

Assessment History

Total Attempts:6

Highest Score:100 %

View last resultsView highest resultRetake quiz

Your result71%

What enzyme converts androstenedione to estradiol?

Your answer:

5-alpha reductase

View correct answer

Correct answer:

CYP19 aromatase
The correct answer is CYP19 aromatase.

What is the primary treatment of hyperthyroidism in pregnancy?

Your answer:

Propylthiouracil

Correct answer:

Propylthiouracil

The correct answer is propylthiouracil.

What is the most common cause of hyperthyroidism in the non-pregnant


woman?

Your answer:

Graves disease

Correct answer:
Graves disease

The correct answer is Graves disease.

What is the definition of premature menopause?

Your answer:

Menopause before 40 years of age

Correct answer:

Menopause before 40 years of age

The correct answer is menopause before 40 years of age.

What is the most common cause of delayed puberty?

Your answer:

Constitutional delay
Correct answer:

Constitutional delay

The correct answer is constitutional delay.

What is the definition of puberty in women?

Your answer:

Physical maturation whereby the women becomes capable of sexual


reproduction

Correct answer:

Physical maturation whereby the women becomes capable of sexual


reproduction

The correct answer is physical maturation whereby the women becomes


capable of sexual reproduction.

Which three hormones consist of almost identical alpha chains?

Your answer:
HCG, FSH, LH

Correct answer:

HCG, FSH, LH

The correct answer is HCG, FSH and LH.

What hormone is secreted by acidophil cells in the anterior pituitary?

Your answer:

Growth hormone

Correct answer:

Growth hormone

The answer is growth hormone.

What hormone is significantly increased on day 0 of the menstrual cycle?


Your answer:

FSH

Correct answer:

FSH

The correct answer is FSH.

With regard to the ovary, what cells produce testosterone?

Your answer:

Leydig

View correct answer

Correct answer:

Theca

The correct answer is theca cells.


What is the finite life span of the corpus luteum?

Your answer:

8 days

View correct answer

Correct answer:

14 days

The correct answer is 14 days.

How many hours after the LH surge does ovulation occur?

Your answer:

24–36 hours

Correct answer:
24–36 hours

The correct answer is 24–36 hours.

Where in the body is calcidiol produced?

Your answer:

Kidneys

View correct answer

Correct answer:

Liver

The correct answer is the liver.

A 16-year-old primiparous woman presents to the Early Pregnancy Unit with a


two-day history of PV bleeding. She is approximately 7 weeks in to a
spontaneous unplanned pregnancy. She is otherwise fit and well. Her serum
βhCG is 94 000 iu/l. During pregnancy, from what structure is βhCG produced?

Your answer:

Syncytiotrophoblast
Correct answer:

Syncytiotrophoblast

The best answer is syncytiotrophoblast.

Add/Edit reflective notes

< Previous pageNext page >

Royal College of Obstetricians and Gynaecologists

© 2018

Registered charity no. 213280

27 Sussex Place
Regent's Park
London NW1 4RG
UK

Tel +44 20 7772 6200


Fax +44 20 7723 0575

Contact Us

Accessibility

Privacy & cookie policy

Terms & conditions

facebook

twitter

linkedin

google
youtube

vimeo

Skip to main content

Toggle navigation

Search StratOG

Home

Exam preparation

Part 1 MRCOG

SBAs for Part 1 MRCOG online resource

Endocrinology page 2
Tutorial history

Progress

100% complete

Submit

Endocrinology page 2

Assessment History

Total Attempts:6

Highest Score:100 %

View last resultsView highest resultRetake quiz

Your result71%

What enzyme converts androstenedione to estradiol?

Your answer:
5-alpha reductase

View correct answer

Correct answer:

CYP19 aromatase

The correct answer is CYP19 aromatase.

What is the primary treatment of hyperthyroidism in pregnancy?

Your answer:

Propylthiouracil

Correct answer:

Propylthiouracil

The correct answer is propylthiouracil.


What is the most common cause of hyperthyroidism in the non-pregnant
woman?

Your answer:

Graves disease

Correct answer:

Graves disease

The correct answer is Graves disease.

What is the definition of premature menopause?

Your answer:

Menopause before 40 years of age

Correct answer:

Menopause before 40 years of age

The correct answer is menopause before 40 years of age.


What is the most common cause of delayed puberty?

Your answer:

Constitutional delay

Correct answer:

Constitutional delay

The correct answer is constitutional delay.

What is the definition of puberty in women?

Your answer:

Physical maturation whereby the women becomes capable of sexual


reproduction

Correct answer:

Physical maturation whereby the women becomes capable of sexual


reproduction
The correct answer is physical maturation whereby the women becomes
capable of sexual reproduction.

Which three hormones consist of almost identical alpha chains?

Your answer:

HCG, FSH, LH

Correct answer:

HCG, FSH, LH

The correct answer is HCG, FSH and LH.

What hormone is secreted by acidophil cells in the anterior pituitary?

Your answer:

Growth hormone

Correct answer:

Growth hormone
The answer is growth hormone.

What hormone is significantly increased on day 0 of the menstrual cycle?

Your answer:

FSH

Correct answer:

FSH

The correct answer is FSH.

With regard to the ovary, what cells produce testosterone?

Your answer:

Leydig

View correct answer


Correct answer:

Theca

The correct answer is theca cells.

What is the finite life span of the corpus luteum?

Your answer:

8 days

View correct answer

Correct answer:

14 days

The correct answer is 14 days.

How many hours after the LH surge does ovulation occur?


Your answer:

24–36 hours

Correct answer:

24–36 hours

The correct answer is 24–36 hours.

Where in the body is calcidiol produced?

Your answer:

Kidneys

View correct answer

Correct answer:

Liver

The correct answer is the liver.

A 16-year-old primiparous woman presents to the Early Pregnancy Unit with a


two-day history of PV bleeding. She is approximately 7 weeks in to a
spontaneous unplanned pregnancy. She is otherwise fit and well. Her serum
βhCG is 94 000 iu/l. During pregnancy, from what structure is βhCG produced?

Your answer:

Syncytiotrophoblast

Correct answer:

Syncytiotrophoblast

The best answer is syncytiotrophoblast.

Add/Edit reflective notes

< Previous pageNext page >

Royal College of Obstetricians and Gynaecologists

© 2018

Registered charity no. 213280

27 Sussex Place
Regent's Park
London NW1 4RG
UK

Tel +44 20 7772 6200

Fax +44 20 7723 0575

Contact Us

Accessibility

Privacy & cookie policy

Terms & conditions

facebook

twitter

linkedin
google

youtube

vimeo


[26/06, 8:49 am] +92 324 2921513: Skip to main content

Toggle navigation

Search StratOG

Home

Exam preparation

Part 1 MRCOG
SBAs for Part 1 MRCOG online resource

Epidemiology / Statistics

Tutorial history

Progress

100% complete

Submit

Epidemiology / Statistics

Assessment History

Total Attempts:4

Highest Score:100 %

View last resultsView highest resultRetake quiz

25 pregnant women were sampled during the antenatal period, looking at the
number of episodes of urinary incontinence. This sample was from a
population of 3500. The number of episodes of incontinence per patient was
30, with a standard deviation of 10. What is the standard error of the mean?

10

If the mean booking weight of an antenatal population of 125 women was 80


kg and the standard error of the mean was 5 kg, what is the 95% confidence
interval?

40-120 kg

60-100 kg

65-95 kg

70-90 kg

75-85 kg
Diagnostic studies frequently use a receiver operating characteristic curve.
What do the two axes of the curve represent?

Accuracy versus 1/sensitivity

Positive likelihood ratio versus negative likelihood ratio

Positive predictive value versus negative predictive value

Sensitivity versus 1 – specificity

Sensitivity versus positive predictive value

If TP = true positive and FP = false positive, what is represented by the


equation TP / (TP + FP)?

Likelihood ratio

Negative predictive value

Positive predictive value

Sensitivity

Specificity
You wish to investigate the time it takes to perform a caesarean section on
women with and without pre-eclampsia. At the end of the study you have two
groups of women with the duration of caesarean sections recorded for each. It
can't be assumed that these are normally distributed. Which non-parametric
statistic test would you use?

Chi-squared test

Linear regression analysis

Mann-Whitney U test

Pearson’s R test

Student’s t-test

In appraising a study, it is important to assess the suitability of the research,


design and methods used in the context of a clinical condition. Which study
will provide the best evidence for a diagnostic test?

Case reports

Cohort study

Cost-benefit study

Cross-sectional study
Randomised controlled trial

Using the data in the table below, what is the likelihood ratio for an ultrasound
Doppler test for deep vein thrombosis (DVT)?

Number of DVT casesNumber of non-DVT casesPositive90 (a)5 (b)Negative10


(c)95 (d)

0.9

10

13.5

18

20

Which statistical test requires a normal population distribution?

Analysis of variance

Chi-squared test

Mann-Whitney U test

Spearman’s rank correlation


Wilcoxon rank-sum test

In a proposed clinical trial of a new uterine relaxant drug, what would be


associated with an increase in the sample size?

A decrease in the specified power of study

A decrease in the standard deviation of blood pressure measurements

A decrease in the type II error rate

An increase in the difference in the outcomes to be detected

An increase in the significant level from 1–5%

Squamous cell carcinoma accounts for approximately how many cases of vulval
cancer?

10%

30%

50%

70%
90%

A woman with a previous caesarean section arrives at term to your unit in


spontaneous labour. The midwife reports the cervix is 6 cm dilated, there is a
longitudinal lie and the vertex is well applied to the cervix. The woman is
contracting three times in 10 minutes. What would you tell this woman her risk
of scar rupture was?

1 in 50

1 in 100

1 in 1000

2 in 1000

2 in 10 000

This is the last question of this quiz. Press finish to submit your choice(s) and
reveal the answer(s).

Finish

Add/Edit reflective notes

< Previous pageNext page >

Royal College of Obstetricians and Gynaecologists


© 2018

Registered charity no. 213280

27 Sussex Place
Regent's Park
London NW1 4RG
UK

Tel +44 20 7772 6200

Fax +44 20 7723 0575

Contact Us

Accessibility

Privacy & cookie policy

Terms & conditions

facebook
twitter

linkedin

google

youtube

vimeo


[26/06, 8:49 am] +92 324 2921513: Skip to main content

Toggle navigation

Search StratOG
Home

Exam preparation

Part 1 MRCOG

SBAs for Part 1 MRCOG online resource

Physiology page 1

Tutorial history

Progress

100% complete

Submit

Physiology page 1

Assessment History

Total Attempts:4
Highest Score:100 %

View last resultsView highest resultRetake quiz

In the process of fertilisation, what is the correct sequence for sperm


penetration into the oocyte?

Corona radiata, zona pellucida, perivitelline space, plasma membrane

Corona radiata, zona pellucida, plasma membrane, perivitelline space

Zona pellucida, corona radiata, plasma membrane, perivitelline space

Zona pellucida, perivitelline space, corona radiata, perivitelline space

Zona pellucida, perivitelline space, plasma membrane, corona radiata

With regard to cardiovascular changes in pregnancy, what are the physiological


changes by the third trimester?

20% increase in blood volume, 35% increase in plasma volume and 10–20%
increase in red blood cell volume

20% increase in blood volume, 45% increase in plasma volume and 20–30%
increase red blood cell volume
30% increase in blood volume, 35% increase in plasma volume and 20–30%
increase red blood cell volume

30% increase in blood volume, 45% increase in plasma volume and 20–30%
increase red blood cell volume

40% increase in blood volume, 35% increase in plasma volume and 30–40%
increase red blood cell volume

There are different types of fetal haemoglobin produced throughout the


gestational period. What is the primary haemoglobin type at 32 weeks of
gestation?

Hb A

Hb F

Hb Gower 1

Hb Gower 2

Hb Portland

Fetal urine is an important contributor to the amniotic fluid volume. At what


gestation does fetal urine become the major contributor to amniotic fluid
volume?

10 weeks
12 weeks

14 weeks

16 weeks

18 weeks

The required daily intake of which vitamins and minerals is not increased in
pregnancy or lactation?

Folic acid

Iron

Vitamin A

Vitamin C

Vitamin D

At what gestational age does the formation of the definitive alveoli take place?

28 weeks

30 weeks
32 weeks

34 weeks

36 weeks

Which lung volume is decreased in pregnancy?

Forced expiratory volume

Functional residual capacity

Peak flow rate

Tidal volume

Vital capacity

Which test is used to diagnose haemolytic anaemia in a newborn?

Cord haemoglobin levels

Cord serum bilirubin levels

Direct Coombs test


Indirect Coombs test

Infection screening test

Signs of hypoglycaemia and hypocalcaemia are often similar. Which sign will
help differentiate the two in a newborn?

Cyanosis

High-pitched cry

Increased muscle tone

Jitteriness

Seizures

The physiological changes in pregnancy are associated with a change in serum


concentration of various hormones. Which test is most appropriate in
diagnosing the thyroid dysfunction in pregnancy?

Free thyroxine (T4) levels

Serum iodine levels

Serum triiodothyronine levels


Thyroid-binding globulin levels

Thyroid-stimulating hormone

A woman presents to the antenatal clinic at 30 weeks of gestation with vulval


varicosities. What is the most likely cause?

Decreased vascular resistance

Increased plasma volume

Increased cardiac output

Infection

Pressure effect of the gravid uterus on the inferior vena cava

What is the normal circulatory life span on a red blood cell?

20 days

40 days

80 days

120 days
160 days

What is the average lifespan of platelets?

5–10 days

10–20 days

40–80 days

80–120 days

120–160 days

What is the physiological mechanism for bile salts in pregnancy?

Aids the absorption of essential amino acids

Aids the absorption of fats

Helps to conjugate bilirubin

Helps to conjugate biliverdin

No known physiological function


This is the last question of this quiz. Press finish to submit your choice(s) and
reveal the answer(s).

Finish

Add/Edit reflective notes

< Previous pageNext page >

Royal College of Obstetricians and Gynaecologists

© 2018

Registered charity no. 213280

27 Sussex Place
Regent's Park
London NW1 4RG
UK

Tel +44 20 7772 6200

Fax +44 20 7723 0575

Contact Us
Accessibility

Privacy & cookie policy

Terms & conditions

facebook

twitter

linkedin

google

youtube
vimeo

Skip to main content

Toggle navigation

Search StratOG

Home

Exam preparation

Part 1 MRCOG

SBAs for Part 1 MRCOG online resource

Physiology page 1

Tutorial history

Progress
100% complete

Submit

Physiology page 1

Assessment History

Total Attempts:4

Highest Score:100 %

View last resultsView highest resultRetake quiz

In the process of fertilisation, what is the correct sequence for sperm


penetration into the oocyte?

Corona radiata, zona pellucida, perivitelline space, plasma membrane

Corona radiata, zona pellucida, plasma membrane, perivitelline space

Zona pellucida, corona radiata, plasma membrane, perivitelline space

Zona pellucida, perivitelline space, corona radiata, perivitelline space


Zona pellucida, perivitelline space, plasma membrane, corona radiata

With regard to cardiovascular changes in pregnancy, what are the physiological


changes by the third trimester?

20% increase in blood volume, 35% increase in plasma volume and 10–20%
increase in red blood cell volume

20% increase in blood volume, 45% increase in plasma volume and 20–30%
increase red blood cell volume

30% increase in blood volume, 35% increase in plasma volume and 20–30%
increase red blood cell volume

30% increase in blood volume, 45% increase in plasma volume and 20–30%
increase red blood cell volume

40% increase in blood volume, 35% increase in plasma volume and 30–40%
increase red blood cell volume

There are different types of fetal haemoglobin produced throughout the


gestational period. What is the primary haemoglobin type at 32 weeks of
gestation?

Hb A

Hb F

Hb Gower 1
Hb Gower 2

Hb Portland

Fetal urine is an important contributor to the amniotic fluid volume. At what


gestation does fetal urine become the major contributor to amniotic fluid
volume?

10 weeks

12 weeks

14 weeks

16 weeks

18 weeks

The required daily intake of which vitamins and minerals is not increased in
pregnancy or lactation?

Folic acid

Iron

Vitamin A
Vitamin C

Vitamin D

At what gestational age does the formation of the definitive alveoli take place?

28 weeks

30 weeks

32 weeks

34 weeks

36 weeks

Which lung volume is decreased in pregnancy?

Forced expiratory volume

Functional residual capacity

Peak flow rate

Tidal volume
Vital capacity

Which test is used to diagnose haemolytic anaemia in a newborn?

Cord haemoglobin levels

Cord serum bilirubin levels

Direct Coombs test

Indirect Coombs test

Infection screening test

Signs of hypoglycaemia and hypocalcaemia are often similar. Which sign will
help differentiate the two in a newborn?

Cyanosis

High-pitched cry

Increased muscle tone

Jitteriness

Seizures
The physiological changes in pregnancy are associated with a change in serum
concentration of various hormones. Which test is most appropriate in
diagnosing the thyroid dysfunction in pregnancy?

Free thyroxine (T4) levels

Serum iodine levels

Serum triiodothyronine levels

Thyroid-binding globulin levels

Thyroid-stimulating hormone

A woman presents to the antenatal clinic at 30 weeks of gestation with vulval


varicosities. What is the most likely cause?

Decreased vascular resistance

Increased plasma volume

Increased cardiac output

Infection

Pressure effect of the gravid uterus on the inferior vena cava


What is the normal circulatory life span on a red blood cell?

20 days

40 days

80 days

120 days

160 days

What is the average lifespan of platelets?

5–10 days

10–20 days

40–80 days

80–120 days

120–160 days

What is the physiological mechanism for bile salts in pregnancy?


Aids the absorption of essential amino acids

Aids the absorption of fats

Helps to conjugate bilirubin

Helps to conjugate biliverdin

No known physiological function

This is the last question of this quiz. Press finish to submit your choice(s) and
reveal the answer(s).

Finish

Add/Edit reflective notes

< Previous pageNext page >

Royal College of Obstetricians and Gynaecologists

© 2018

Registered charity no. 213280

27 Sussex Place
Regent's Park
London NW1 4RG
UK

Tel +44 20 7772 6200

Fax +44 20 7723 0575

Contact Us

Accessibility

Privacy & cookie policy

Terms & conditions

facebook

twitter

linkedin
google

youtube

vimeo


[26/06, 8:49 am] +92 324 2921513: Skip to main content

Toggle navigation

Search StratOG

Home

Exam preparation

Part 1 MRCOG
SBAs for Part 1 MRCOG online resource

Physiology page 2

Tutorial history

Progress

100% complete

Submit

Physiology page 2

Assessment History

Total Attempts:5

Highest Score:100 %

View last resultsView highest resultRetake quiz

With regard to gestational diabetes, what is the likely change in plasma free
fatty acids (FFA) and plasma ketone bodies?
Decreased FFA and decreased ketone bodies

Increased FFA and decreased ketone bodies

Increased FFA and increased ketone bodies

Increased FFA and no change in ketone bodies

No change in FFA and increased ketone bodies

What cells are transformed by spermeogenesis?

Primary spermatocytes

Secondary spermatocytes

Spermatids

Spermatogonia

Tertiary spermatocysts

How many times the normal range is alkaline phosphatase increased in


pregnancy?

x2
x3

x5

x10

x12

At what gestation is the physiological dilution of anaemia the greatest?

12 weeks of gestation

24 weeks of gestation

32 weeks of gestation

38 weeks of gestation

42 weeks of gestation

What is the increase in oxygen consumption in pregnancy?

10 ml/min

20 ml/min
30 ml/min

40 ml/min

50 ml/min

What lung function test volume is increased in pregnancy?

Expiratory reserve volume

Inspiratory reserve volume

Respiratory dead volume

Tidal volume

Total lung capacity

What is the change in FEV1 in pregnancy?

–20%

–10%

No change
+10%

+20%

What is the change in total lung capacity in pregnancy?

Decreased by 100 ml

Decreased by 200 ml

Increased by 100 ml

Increased by 200 ml

No change

What proportion of body calcium is stored in the skeletal system?

5%

10%

80%

90%
99%

What is the WHO definition of osteoporosis?

A spinal fracture or fracture of the hip with low calcium

T score < –1.0

T score < –2.5

Z score < –1.0

Z score < –2.5

What happens to gallbladder motility and bile composition in pregnancy?

Decreases gallbladder motility and decreases the lithogenicity of bile

Decreases gallbladder motility and increases the lithogenicity of bile

Has no effect on gallbladder motility and the lithogenicity of bile

Increases gallbladder motility and decreases the lithogenicity of bile

Increases gallbladder motility and increases the lithogenicity of bile


Regarding rheumatoid arthritis in pregnancy, which of the following is correct?

Epidural analgesia is relatively contraindicated

Exacerbation tends to occur in puerperium

Folic acid in high dose is beneficial

Methotrexate is safe to use in the second and third trimester

Usually exacerbates around 28–32 weeks of gestation

A primiparous 18-year-old woman presents to the maternity assessment unit


at 39 weeks of gestation. Urinalysis reveals a proteinuria value of 3+. The
woman has a blood pressure of 170/110 mmHg and she is suffering from a
headache. What underlying pathology would brisk reflexes indicate?

A stroke

Alcohol intoxication

Cerebral irritation

Hypothyroid disease

Lower motor neurone disease


A primiparous woman with a BMI of 40 from South East Asia is seen by you in
antenatal clinic. The woman is at 24 weeks of gestation and has a fundal height
of 30 cm. (Same scenario as above.) What vitamin would you prescribe for this
woman?

Vitamin A

Vitamin B6

Vitamin B12

Vitamin C

Vitamin D

A primiparous woman with a BMI of 40 from South East Asia is seen by you in
antenatal clinic. The woman is at 24 weeks of gestation and has a fundal height
of 30 cm. What initial investigation do you organise?

24-hour urine collection

Glucose tolerance test.

Liver function tests

Referral to the dietician


Serum cholesterol

This is the last question of this quiz. Press finish to submit your choice(s) and
reveal the answer(s).

Finish

Add/Edit reflective notes

< Previous pageNext page >

Royal College of Obstetricians and Gynaecologists

© 2018

Registered charity no. 213280

27 Sussex Place
Regent's Park
London NW1 4RG
UK

Tel +44 20 7772 6200

Fax +44 20 7723 0575

Contact Us
Accessibility

Privacy & cookie policy

Terms & conditions

facebook

twitter

linkedin

google

youtube
vimeo


[26/06, 8:49 am] +92 324 2921513: Skip to main content

Toggle navigation

Search StratOG

Home

Exam preparation

Part 1 MRCOG

SBAs for Part 1 MRCOG online resource

Physiology page 2

Tutorial history

Progress
100% complete

Submit

Physiology page 2

Assessment History

Total Attempts:6

Highest Score:100 %

View last resultsView highest resultRetake quiz

Your result40%

With regard to gestational diabetes, what is the likely change in plasma free
fatty acids (FFA) and plasma ketone bodies?

Your answer:

Increased FFA and increased ketone bodies

Correct answer:
Increased FFA and increased ketone bodies

The correct answer is increased FFA and increased ketone bodies.

What cells are transformed by spermeogenesis?

Your answer:

Spermatids

Correct answer:

Spermatids

The correct answer is spermatids.

How many times the normal range is alkaline phosphatase increased in


pregnancy?

Your answer:

x2
View correct answer

At what gestation is the physiological dilution of anaemia the greatest?

Your answer:

12 weeks of gestation

View correct answer

What is the increase in oxygen consumption in pregnancy?

Your answer:

20 ml/min

View correct answer

What lung function test volume is increased in pregnancy?

Your answer:

Inspiratory reserve volume

View correct answer


What is the change in FEV1 in pregnancy?

Your answer:

No change

Correct answer:

No change

The correct answer is no change.

What is the change in total lung capacity in pregnancy?

Your answer:

No change

View correct answer

What proportion of body calcium is stored in the skeletal system?

Your answer:
10%

View correct answer

What is the WHO definition of osteoporosis?

Your answer:

T score < –2.5

Correct answer:

T score < –2.5

The correct answer is a T score < –2.5 .

CategoryT-score range% young womenNormalT-score ≥ –1.085%OsteopeniaT-


score between –2.5 and –1.014%OsteoperosisT-score ≤ –2.50.6%Severe
osteoperosisT-score ≤ –2.5 wth fragility fracture

What happens to gallbladder motility and bile composition in pregnancy?

Your answer:

Increases gallbladder motility and decreases the lithogenicity of bile


View correct answer

Regarding rheumatoid arthritis in pregnancy, which of the following is correct?

Your answer:

Exacerbation tends to occur in puerperium

Correct answer:

Exacerbation tends to occur in puerperium

The correct answer is exacerbation tends to occur in puerperium.

A primiparous 18-year-old woman presents to the maternity assessment unit


at 39 weeks of gestation. Urinalysis reveals a proteinuria value of 3+. The
woman has a blood pressure of 170/110 mmHg and she is suffering from a
headache. What underlying pathology would brisk reflexes indicate?

Your answer:

A stroke

View correct answer


A primiparous woman with a BMI of 40 from South East Asia is seen by you in
antenatal clinic. The woman is at 24 weeks of gestation and has a fundal height
of 30 cm. (Same scenario as above.) What vitamin would you prescribe for this
woman?

Your answer:

Vitamin B6

View correct answer

A primiparous woman with a BMI of 40 from South East Asia is seen by you in
antenatal clinic. The woman is at 24 weeks of gestation and has a fundal height
of 30 cm. What initial investigation do you organise?

Your answer:

Glucose tolerance test.

Correct answer:

Glucose tolerance test.

The best answer is glucose tolerance test.


Add/Edit reflective notes

< Previous pageNext page >

Royal College of Obstetricians and Gynaecologists

© 2018

Registered charity no. 213280

27 Sussex Place
Regent's Park
London NW1 4RG
UK

Tel +44 20 7772 6200

Fax +44 20 7723 0575

Contact Us

Accessibility

Privacy & cookie policy

Terms & conditions


facebook

twitter

linkedin

google

youtube

vimeo


[26/06, 8:49 am] +92 324 2921513: Skip to main content
Toggle navigation

Search StratOG

Home

Exam preparation

Part 1 MRCOG

SBAs for Part 1 MRCOG online resource

Biophysics

Tutorial history

Progress

100% complete

Submit
Biophysics

Assessment History

Total Attempts:5

Highest Score:100 %

View last resultsView highest resultRetake quiz

Your result94%

What is the normal axis of the heart?

Your answer:

–30 to +90 degrees

Correct answer:

–30 to +90 degrees

The answer is Minus 30 to plus 90 degrees. In cases of right ventricular


hypertrophy the axis shifts to the right and there is left shift with left
ventricular hypertrophy.

What is the frequency of a transvaginal ultrasound scan?


Your answer:

6 MHz

Correct answer:

6 MHz

The answer is 6 MHz. The higher the frequency the greater the axial resolution
of ultrasound meaning you can distinguish between structures based on their
distance from the ultrasound beam). Unfortunately, increasing the frequency
reduces the penetration of the beam. Transabdominal ultrasound therefore
requires a lower frequency and has a lower resolution. Lateral resolution
(distinguishing two structures that are the same distance away from the
ultrasound beam but are side by side) is not improved by increasing the
frequency and can only be increased by improving the focus of the beam.

In a Dexa bone mineral density scan the results are reported which two ways?

Your answer:

T score (compares the individual to a young adult where normal is greater than
–1) and
Z score (compares the individual to another individual of the same age and
gender)

Correct answer:
T score (compares the individual to a young adult where normal is greater than
–1) and
Z score (compares the individual to another individual of the same age and
gender)

The answer is T score (compares the individual to a young adult where normal
is greater than –1) and Z score (compares the individual to another individual
of the same age and gender).

A T score between +1 and –1 indicates a normal bone (since it is compared to a


younger population it is likely to be a negative cut off). A patient with a score
between –1 and –2.5 is osteopenic. A score of less then –2.5 is osteoporotic. A
Z score of less than -1 is suggestive of factors other than aging such as thyroid
problems, malnutrition, medications etc contributing to the osteoporosis.

You wish to achieve a cutting and simultaneous coagulation effect from the
diathermy machine. How is this best achieved?

Your answer:

Use monopolar diathermy with blended waveform

Correct answer:

Use monopolar diathermy with blended waveform

The answer is Use monopolar diathermy with blended waveform. Bipolar can
only coagulate; it cannot cut and coagulate effectively.
In monopolar diathermy which feature prevents electrocution occurring?

Your answer:

High frequency

Correct answer:

High frequency

The answer is High frequency. The high frequency prevents depolarisation of


the cell. The voltage and current would certainly be enough for electrocution
to occur.

With regard to ultrasound with colour Doppler, how can the blue coloured
flow pattern be described?

Your answer:

Flow away from the colour box

Correct answer:

Flow away from the colour box

The answer is Flow away from the colour box. The colour only describes the
flow of the fluid towards (red) or away (blue) from the colour box. It does not
correlate to venous or arterial flow.
Which laser can be used in gynaecological surgery, is invisible without an aim
beam and cannot be transmitted down a fibre optic cable?

Your answer:

Carbon dioxide

Correct answer:

Carbon dioxide

The answer is Carbon dioxide. The CO2 laser generates an invisible light and
requires an aiming beam, usually helium–neon. It cannot be transmitted down
a fibre optic cable and is absorbed by water. It is used in cervical treatment but
is limited by the fact that it does not work in the presence of blood.

At the end of surgery you notice an area of burn near the attached diathermy
plate. The diathermy plate seems to be partially detached. Which of the
following would be the most likely aetiology?

Your answer:

Due to the plate not being secured properly, the current density was
concentrated in the smaller area where the plate was attached

Correct answer:
Due to the plate not being secured properly, the current density was
concentrated in the smaller area where the plate was attached

The answer is Due to the plate not being secured properly, the current density
was concentrated in the smaller area where the plate was attached. The
amount of thermal damage is dependent upon the current density and the size
of the current. The current density in the tip of the needle will be very high as
the current is concentrated in the small point. The plate used for the return
electrode in monopolar diathermy has a larger surface area of contact,
resulting in a much lower current density. Any thermal effect is therefore
widely dissipated. This highlights the need for the whole surface of the plate to
be securely attached to the body.

What would be the frequency of the electric current used in electrosurgery?

Your answer:

10 000 Hz

View correct answer

Correct answer:

500 Hz

The answer is 500 Hz. When an electric current of 50 Hz (household frequency)


passes through the body it causes an irreversible depolarisation of the cell
membrane. It may also cause depolarisation of cardiac muscles, resulting into
death. If the current frequency is increased above 200 Hz depolarisation does
not occur. Instead ions are excited to produce thermal effect. A frequency of
100 Hz may cause neuromuscular stimulation. The AM radio band utilises the
frequency of 10,000 Hz.

With regard to electrosurgery, if carbon is seen on the tip of an electrode the


surgeon can assume that, at some stage in the operation, what temperature
has been reached?

Your answer:

200°C

Correct answer:

200°C

The answer is 200°C. Necrosis occurs at 44°C. Coagulation occurs at 70°C.


Desiccation occurs at 90°C. Carbonisation occurs at 200°C.

Optimum intra-abdominal pressure should be achieved to insert the primary


trocar safely at laparoscopy. What is the optimum pressure range?

Your answer:

20–25 mmHg

Correct answer:

20–25 mmHg
The answer is 20–25 mmHg. An intra-abdominal pressure of 20–25 mmHg
results in increasing splinting. This allows the trocar to move more easily
through the abdominal layers. No adverse effect on respiratory function is
reported using such a pressure range. Once the primary trocar is inserted, the
secondary trocar can be visually introduced and the pressure can be lowered.

You have been asked to review an early pregnancy ultrasound scan that shows
a mean gestational sac of less than 25 mm and a crown rump length less than 6
mm with no visible fetal heart. What is the likely diagnosis?

Your answer:

Pregnancy of uncertain viability

Correct answer:

Pregnancy of uncertain viability

The answer is Pregnancy of uncertain viability. This woman should be


reassessed in 7 to 10 days time to assess viability, according to RCOG criteria. If
the gestational sac is larger than 25 mm with no fetal pole, it is suggestive of
silent miscarriage. Individual hospital guidelines vary. Pregnancy of unknown
location is a condition where serum beta-hCG is above the discriminatory zone
for ultrasound detection, but no intra or extrauterine pregnancy is detected on
the scan.

In the case of hypocalcaemia, what electrocardiography (ECG) changes


typically occur?
Your answer:

Long QT interval

Correct answer:

Long QT interval

The answer is Long QT interval. This is clinically important because it can result
in a cardiac arrest. Long QT syndrome can have a genetic basis, both with
recessive and dominant modes of transmission. There are several drugs that
can cause a long QT interval including haloperidol, erythromycin and
terfenadine.

With regard to the jugular venous pressure (JVP) waveform, what can cause an
elevated JVP reading with a normal waveform?

Your answer:

Right-sided heart failure

Correct answer:

Right-sided heart failure

The answer is Right-sided heart failure. Although tricuspid valve disorders can
cause an elevated JVP, there will not be a normal wave form.
A 53-year-old woman presents with chronic renal failure and a potassium
concentration of 5.8 mmol/l. An electrocardiography (ECG) has been
requested. What ECG feature can occur with this potassium concentration?

Your answer:

Tall T waves, wide QRS complex

Correct answer:

Tall T waves, wide QRS complex

The answer is Tall T waves, wide QRS complex. These are important to note
because sudden cardiac arrest can occur if corrective action is not taken.

A 16-year-old primiparous woman presents to the Early Pregnancy Unit with a


two-day history of PV bleeding. She is approximately 7 weeks in to a
spontaneous unplanned pregnancy. She is otherwise fit and well. Her serum
HCG is 94 000 iu/l. An ultrasound scan is performed on this patient which
shows no evidence of a fetal pole, but does show a placental mass in the
uterus, containing multiple echoes with a 'bunch of grapes' appearance. These
ultrasound findings are characteristic of which condition?

Your answer:

Complete hydatidiform mole

Correct answer:
Complete hydatidiform mole

The best answer is complete hydatidiform mole.

Add/Edit reflective notes

< Previous pageNext page >

Royal College of Obstetricians and Gynaecologists

© 2018

Registered charity no. 213280

27 Sussex Place
Regent's Park
London NW1 4RG
UK

Tel +44 20 7772 6200

Fax +44 20 7723 0575

Contact Us
Accessibility

Privacy & cookie policy

Terms & conditions

facebook

twitter

linkedin

google

youtube
vimeo


[26/06, 8:49 am] +92 324 2921513: Skip to main content

Toggle navigation

Search StratOG

Home

Exam preparation

Part 1 MRCOG

SBAs for Part 1 MRCOG online resource

Biophysics

Tutorial history

Progress
100% complete

Submit

Biophysics

Assessment History

Total Attempts:4

Highest Score:100 %

View last resultsView highest resultRetake quiz

What is the normal axis of the heart?

–0 to +90 degrees

–20 to +70 degrees

–30 to +90 degrees

–90 to +180 degrees


Zero to +90 degrees

What is the frequency of a transvaginal ultrasound scan?

2 MHz

6 MHz

18 MHz

23 MHz

32 MHz

In a Dexa bone mineral density scan the results are reported which two ways?

T score (compares the individual to a young adult where normal is greater than
–1) and
Z score (compares the individual to another individual of the same age and
gender)

T score (compares the individual to a young adult where normal is greater than
0) and
Z score (compares the individual to another individual of the same age and
gender)

Z score (compares the individual to a young adult where normal is greater than
-1) and
T score (compares the individual to another individual of the same age and
gender)

Z score (compares the individual to a young adult where normal is greater than
0) and
T score (compares the individual to another individual of the same age and
gender)

Z score (compares the individual to a young adult where normal is greater than
–2.5) and
T score (compares the individual to another individual of the same age and
gender)

You wish to achieve a cutting and simultaneous coagulation effect from the
diathermy machine. How is this best achieved?

Use bipolar diathermy with blended waveform

Use bipolar diathermy with yellow (cutting pedal)

Use fulguration

Use monopolar diathermy with blended waveform

Use monopolar diathermy with yellow (cutting) and blue (coagulation) pedal
simultaneously

In monopolar diathermy which feature prevents electrocution occurring?


Direct current (DC) not alternative current (AC)

High frequency

Low current

Low voltage

The patient is earthed

With regard to ultrasound with colour Doppler, how can the blue coloured
flow pattern be described?

Deoxygenated blood flow

Flow away from the colour box

Flow towards the colour box

Low volume of flow per unit area

Venous flow pattern

Which laser can be used in gynaecological surgery, is invisible without an aim


beam and cannot be transmitted down a fibre optic cable?
Argon

Carbon dioxide

Helium–neon

Neodymium:YAG

Potassium titanyl phosphate

At the end of surgery you notice an area of burn near the attached diathermy
plate. The diathermy plate seems to be partially detached. Which of the
following would be the most likely aetiology?

Due to the plate not being secured properly, the current density was
concentrated in the smaller area where the plate was attached

High voltage was used

The injury may have resulted from insulation failure

The power output was accidentally turned up

The surgeon may have accidently touched the instrument to the patient’s thigh

What would be the frequency of the electric current used in electrosurgery?


35 Hz

50 Hz

100 Hz

500 Hz

10 000 Hz

With regard to electrosurgery, if carbon is seen on the tip of an electrode the


surgeon can assume that, at some stage in the operation, what temperature
has been reached?

44°C

70°C

90°C

150°C

200°C

Optimum intra-abdominal pressure should be achieved to insert the primary


trocar safely at laparoscopy. What is the optimum pressure range?
10–15 mmHg

10–15 mm water

15–18 mmHg

20–25 mmHg

20–25 mm water

You have been asked to review an early pregnancy ultrasound scan that shows
a mean gestational sac of less than 25 mm and a crown rump length less than 6
mm with no visible fetal heart. What is the likely diagnosis?

Inevitable miscarriage

Missed miscarriage

Pregnancy of uncertain viability

Pregnancy of unknown location

Threatened miscarriage

In the case of hypocalcaemia, what electrocardiography (ECG) changes


typically occur?
Long QT interval

Short QT interval

ST depression

ST elevation

The presence of Q waves

With regard to the jugular venous pressure (JVP) waveform, what can cause an
elevated JVP reading with a normal waveform?

Atrial fibrillation

Left-sided heart failure

Right-sided heart failure

Tricuspid regurgitation

Tricuspid stenosis

A 53-year-old woman presents with chronic renal failure and a potassium


concentration of 5.8 mmol/l. An electrocardiography (ECG) has been
requested. What ECG feature can occur with this potassium concentration?
Flattened T Waves, ST depression

Flattened T waves, ST elevation

Tall T waves, ST depression

Tall T waves, wide QRS complex

Tall U waves, ST depression

A 16-year-old primiparous woman presents to the Early Pregnancy Unit with a


two-day history of PV bleeding. She is approximately 7 weeks in to a
spontaneous unplanned pregnancy. She is otherwise fit and well. Her serum
HCG is 94 000 iu/l. An ultrasound scan is performed on this patient which
shows no evidence of a fetal pole, but does show a placental mass in the
uterus, containing multiple echoes with a 'bunch of grapes' appearance. These
ultrasound findings are characteristic of which condition?

Complete hydatidiform mole

Endometrioma

Live intrauterine pregnancy

Missed miscarriage

Partial hydatidiform mole


This is the last question of this quiz. Press finish to submit your choice(s) and
reveal the answer(s).

Finish

Add/Edit reflective notes

< Previous pageNext page >

Royal College of Obstetricians and Gynaecologists

© 2018

Registered charity no. 213280

27 Sussex Place
Regent's Park
London NW1 4RG
UK

Tel +44 20 7772 6200

Fax +44 20 7723 0575

Contact Us

Accessibility
Privacy & cookie policy

Terms & conditions

facebook

twitter

linkedin

google

youtube

vimeo

[26/06, 8:49 am] +92 324 2921513: Skip to main content

Toggle navigation

Search StratOG

Home

Exam preparation

Part 1 MRCOG

SBAs for Part 1 MRCOG online resource

Clinical management

Tutorial history

Progress
100% complete

Submit

Clinical management

Assessment History

Total Attempts:4

Highest Score:100 %

View last resultsView highest resultRetake quiz

A 16-year-old woman presents with secondary amenorrhoea. She is healthy,


with no past medical history. Her BMI is 17. What is the most appropriate
initial investigation?

Bone mineral density scan

Dehydroepiandrosterone sulphate (DHEAS) measurement

Follicle-stimulating hormone measurement

Karyotype

Thyroid function test


A 55-year-old woman presents to the clinic enquiring about the use of
hormone replacement therapy (HRT). She had a hysterectomy 8 years ago for
fibroids. She has no contraindications or other past medical history, except a
strong family history of osteoporosis. Her main symptoms are hot flushes and
vaginal dryness. What would be your first treatment option?

A selective estrogen receptor modulator

Combined sequential HRT

Estrogen-only HRT

Oral calcium therapy only

Selective serotonin reuptake inhibitor

An 18-year-old woman presents to an early pregnancy unit with light vaginal


bleeding after 10 weeks of amenorrhea. She had taken a pregnancy test 4
weeks ago that had tested positive. A transvaginal ultrasound scan showed an
irregular gestation sac with no fetal pole. What is the likely diagnosis?

Anembryonic pregnancy

Complete miscarriage

Hydatidiform mole
Incomplete miscarriage

Threatened miscarriage

A primigravid woman presents in spontaneous labour at 39 weeks of gestation.


At 18:00h, her cervical dilatation is 6 cm. A further vaginal examination at
22:00h reveals that cervical dilatation is still at 6 cm. At 02:10h, the fetus is in
the occipitoposterior position and uterine activity is present. What is the most
appropriate action?

Amniotomy

Commence intravenous oxytocin

Membrane sweep

Repeat vaginal examination after 2 hours

Repeat vaginal examination after 4 hours

A 17-year-old woman presents to the sexual health clinic with vulval ulceration
and difficulty in passing urine. She is sexually active and has had unprotected
intercourse with her new boyfriend. She takes the combined oral
contraceptive pill. What is the most likely diagnosis?

Candida albicans

Herpes simplex virus


Herpes varicella virus

Human papillomavirus

Syphilis

Whilst you are attending to a woman in the antenatal ward, the


woman collapses and becomes unresponsive. You open their airway but they
are not breathing. What should you do next?

Commence artificial ventilation

Commence cardiac compressions

Get help

Give a precordial thump

Left lateral tilt

You answer an emergency call for a postpartum haemorrhage. The midwife


estimates that the woman has lost approximately 1 l of blood. What should
you be your first action?

Assess the woman’s airway, breathing and circulation and administer oxygen
at a rate of 15 l/min
Bimanual compression of the uterus

Catheterise the bladder

Obtain blood for cross match of 4 units

Site two large bore intravenous cannulae

You answer an emergency call for a postpartum haemorrhage. The midwife


estimates that the woman has lost approximately 500 ml of blood. What is the
most likely cause of the bleeding?

Cervical trauma

Coagulopathy

Retained placenta tissue

Uterine atony

Vaginal tear

A cardiotocograph shows type 1 variable decelerations. What is the cause of


this feature?

Fetal head compression


Fetal hypoxia

Fetal movements

Placental insufficiency

Umbilical cord compression

You are asked to assess a woman who is receiving magnesium sulphate


infusions for severe pre-eclampsia. They have passed only 5 ml urine in the last
2 hours. Tests demonstrate that their deep tendon reflexes are absent. What
other observation should you take?

Blood pressure

Glasgow coma score

Pulse rate

Respiratory rate

Temperature

Polyglactin sutures are used extensively in surgical procedures, particularly to


ligate vessels. What are the key features of polyglactin sutures?

Braided, absorbable and synthetic


Braided, non-absorbable and synthetic

Non-braided, absorbable and natural

Non-braided, absorbable and synthetic

Non-braided, non-absorbable and natural

A nulliparous woman presents with spontaneous rupture of membranes at 41


weeks of gestation. At 18:00h, her cervical dilatation is 3 cm. A further vaginal
examination at 22:00h reveals that her cervical dilatation is still 3 cm. At
02:10h, the fetus is in the occipitoposterior position and uterine activity is
present. What is the most appropriate action?

Administer prostaglandin per vaginam

Caesarean section

Commence intravenous oxytocin

Membrane sweep

Repeat vaginal examination after 4 hours

A 68-year-old woman presents with two episodes of postmenopausal bleeding.


She has a BMI of 23 and is otherwise healthy. An ultrasound shows that her
endometrial cavity is 4 mm thick, and an endometrial pipelle sample is taken
that yields a small volume of tissue. The pathology report suggests a neoplasm.
What is the most likely diagnosis?

Leiomyoma

Endometrial hyperplasia

Endometrial polyp

Endometrioid adenocarcinoma

Serous carcinoma

A 75-year-old woman presents with increased weight, loss of hair and a dislike
of cold weather. She was noted to have a high prolactin and normal
electrolytes. She is otherwise fit and well. What is the likely cause of the high
prolactin?

Antiemetic use

Hypothyroidism

Neuroleptic use

Prolactinoma

Renal failure
The midwife has asked you to review a 34-year-old multigravida who is 7 cm
dilated and progressing well. The CTG shows a variability of <5. She was given
pethidine when she was 6 cm dilated. A fetal blood sampling is done and
reveals a pH of 7.23. What will be your next line of management?

Delivery is indicated

Reassure the woman

Repeat FBS after 1 hour

Repeat FBS if fetal heart rate abnormality persists

Repeat FBS within 30 minutes

A 30-year-old primigravida who is pregnant after IVF attends the Early


Pregnancy Unit with severe right sided abdominal pain. An ultrasound scan
shows an empty uterus. β-hCG levels are 5,500 iu/L. What is the appropriate
course of action?

Expectant management, evaluate after 48 hours with repeat hCG levels

Laparoscopy and salpingectomy if there is an ectopic pregnancy

Laparoscopy and salpingotomy if there is an ectopic pregnancy

Methotrexate injection
Repeat scan in 7 days

A 54-year-old woman presents to her GP with malaise, a history of recent


weight loss and bloating. On examination the GP notes a large pelvic mass. He
sends her for an urgent abdominal ultrasound scan and initiates a 2-week-wait
referral. This woman is discussed at an MDT meeting and it is decided to
proceed with a laparatomy. During the laparotomy, a full examination is
performed to look for possible metastatic spread. If this cancer is presumed to
be ovarian, which lymph nodes would you expect the disease to drain to first?

External iliac lymph nodes

Hypogastric lymph nodes

Inguinal lymph nodes

Internal iliac lymph nodes

Para-aortic lymph nodes

A 28-year-old primiparous woman is induced at 39+2 due to a raised blood


pressure and significant proteinuria. She progresses well to full dilatation but
after pushing for 2 hours there is no vertex visible. A decision is made for an
instrumental delivery, and the baby is delivered via an assisted vaginal delivery
in theatre. The episiotomy extends and the tear involves approximately 60% of
the external anal sphincter. The internal anal sphincter and anal mucosa are
intact. How would you classify this perineal trauma?

Second degree tear


3a degree tear

3b degree tear

3c degree tear

Fourth degree tear

A 28-year-old primiparous woman is induced at 39+2 due to a raised blood


pressure and significant proteinuria. She progresses well to full dilatation but
after pushing for 2 hours there is no vertex visible. On examination, the head is
felt to be in an occiput anterior position at +1 station. A decision is made for an
instrumental delivery, and the baby is delivered via assisted vaginal delivery in
theatre. For a ventouse delivery to be successful, the ventouse cup needs to be
applied to the flexion point of the fetal head. Where on the fetal head is the
flexion point located?

Approximately 3 cm anterior (in front) of the anterior fontanelle

Directly over the anterior fontanelle

Directly over the posterior fontanelle

On the sagittal suture line, approximately 2 cm posterior (behind) the posterior


fontanelle

On the sagittal suture line, approximately 3 cm anterior (in front) of the


posterior fontanelle
A woman with a previous caesarean section arrives at term to your unit in
spontaneous labour. The midwife reports the cervix is 6 cm dilated, there is a
longitudinal lie and the vertex is well applied to the cervix. The woman is
contracting three times in 10 minutes. The contractions stop and there is a
fetal bradycardia. What is the most likely diagnosis?

Placental abruption

Scar dehiscence

Scar rupture

Tetanic uterine contractions

Uterine atony

A woman with a previous caesarean section arrives at term to your unit in


spontaneous labour. The midwife reports the cervix is 6 cm dilated, there is a
longitudinal lie and the vertex is well applied to the cervix. The woman is
contracting three times in 10 minutes. You are asked to assess the woman by
the registrar on call as he is in theatre. What initial management would you
suggest?

You immediately perform a fetal blood sample

You suggest continuous fetal monitoring (CFM) and syntocinon infusion


You suggest continuous fetal monitoring (CFM) and vaginal assessment 2 hours
following the last VE

You suggest intermittent monitoring and 1 hourly vaginal assessments

You suggest mobilisation

A 15-year-old girl visits her GP requesting a method of contraception. When


taking her history, you discover a personal history of thromboembolic events.
She has had three sexual partners in 5 months. She also complains of
dysmenorrhea. You advise on contraception being mindful of which medical
law?

Abortion Act 1967

Bolam

Bolitho

Fraser

Gillick

This is the last question of this quiz. Press finish to submit your choice(s) and
reveal the answer(s).

Finish
Add/Edit reflective notes

< Previous pageNext page >

Royal College of Obstetricians and Gynaecologists

© 2018

Registered charity no. 213280

27 Sussex Place
Regent's Park
London NW1 4RG
UK

Tel +44 20 7772 6200

Fax +44 20 7723 0575

Contact Us

Accessibility

Privacy & cookie policy

Terms & conditions


facebook

twitter

linkedin

google

youtube

vimeo


[26/06, 8:49 am] +92 324 2921513: Skip to main content
Toggle navigation

Search StratOG

Home

Exam preparation

Part 1 MRCOG

SBAs for Part 1 MRCOG online resource

Clinical management

Tutorial history

Progress

100% complete

Submit
Clinical management

Assessment History

Total Attempts:5

Highest Score:100 %

View last resultsView highest resultRetake quiz

Your result86%

A 16-year-old woman presents with secondary amenorrhoea. She is healthy,


with no past medical history. Her BMI is 17. What is the most appropriate
initial investigation?

Your answer:

Follicle-stimulating hormone measurement

Correct answer:

Follicle-stimulating hormone measurement

The answer is follicle-stimulating hormone measurement. The likely diagnosis


is amenorrhoea caused by weight loss, so follicle-stimulating hormone
measurements would be appropriate. Since she is clinically euthyroid, thyroid
function tests would be of limited value.
A 55-year-old woman presents to the clinic enquiring about the use of
hormone replacement therapy (HRT). She had a hysterectomy 8 years ago for
fibroids. She has no contraindications or other past medical history, except a
strong family history of osteoporosis. Her main symptoms are hot flushes and
vaginal dryness. What would be your first treatment option?

Your answer:

Estrogen-only HRT

Correct answer:

Estrogen-only HRT

The answer is estrogen-only HRT. HRT provides the most effective method of
treating climacteric hot flushes and vaginal dryness, as well as postmenopausal
osteoporosis. Estrogen-only HRT is appropriate following a hysterectomy, as
the risk of endometrial carcinoma is not present.

An 18-year-old woman presents to an early pregnancy unit with light vaginal


bleeding after 10 weeks of amenorrhea. She had taken a pregnancy test 4
weeks ago that had tested positive. A transvaginal ultrasound scan showed an
irregular gestation sac with no fetal pole. What is the likely diagnosis?

Your answer:

Anembryonic pregnancy
Correct answer:

Anembryonic pregnancy

The answer is anembryonic pregnancy. Threatened miscarriage refers to


vaginal bleeding in the presence of a viable pregnancy; however, this
pregnancy is not viable. There are products of conception (gestation sac) that
are visible on the scan so this is an incomplete miscarriage, which is usually
associated with heavy vaginal bleeding. In this case, the woman is considered
to be pregnant, although no embryonic tissue is present. Therefore, the most
likely diagnosis is anembryonic pregnancy.

A primigravid woman presents in spontaneous labour at 39 weeks of gestation.


At 18:00h, her cervical dilatation is 6 cm. A further vaginal examination at
22:00h reveals that cervical dilatation is still at 6 cm. At 02:10h, the fetus is in
the occipitoposterior position and uterine activity is present. What is the most
appropriate action?

Your answer:

Amniotomy

Correct answer:

Amniotomy

The answer is amniotomy. This case demonstrates slow/no progression during


the first stage of labour and malpositioning of the fetus. The most appropriate
initial intervention would be an amniotomy (artificial rupture of the
membranes).
A 17-year-old woman presents to the sexual health clinic with vulval ulceration
and difficulty in passing urine. She is sexually active and has had unprotected
intercourse with her new boyfriend. She takes the combined oral
contraceptive pill. What is the most likely diagnosis?

Your answer:

Herpes simplex virus

Correct answer:

Herpes simplex virus

The answer is herpes simplex virus. A painful genital ulcer has developed in a
sexually active patient who has not used barrier contraception. Syphilis is
usually associated with the presence of a painless ulcer. Candida is not a
sexually transmitted disease and rarely presents with ulceration. Human
papillomavirus causes warts, and herpes varicella zoster causes chicken pox
and shingles.

Whilst you are attending to a woman in the antenatal ward, the


woman collapses and becomes unresponsive. You open their airway but they
are not breathing. What should you do next?

Your answer:

Get help
Correct answer:

Get help

The answer is get help. Basic and Advanced Life Support guidelines highlight
the need to get help if a patient has collapsed and is unresponsive. You should
then place the patient in left lateral tilt and commence cardiac compressions at
a rate of 30:2. Artificial ventilation is not mandatory in the ALS guidelines.

You answer an emergency call for a postpartum haemorrhage. The midwife


estimates that the woman has lost approximately 1 l of blood. What should
you be your first action?

Your answer:

Assess the woman’s airway, breathing and circulation and administer oxygen
at a rate of 15 l/min

Correct answer:

Assess the woman’s airway, breathing and circulation and administer oxygen
at a rate of 15 l/min

The answer is assess the woman’s airway, breathing and circulation and
administer oxygen at a rate of 15 l/min. In an acute emergency, one should
always assess the airway, breathing and circulation before addressing the
secondary treatment. In practice, this may be talking to the woman and to see
if they respond.
You answer an emergency call for a postpartum haemorrhage. The midwife
estimates that the woman has lost approximately 500 ml of blood. What is the
most likely cause of the bleeding?

Your answer:

Uterine atony

Correct answer:

Uterine atony

The answer is uterine atony. Approximately 70% of all postpartum


haemorrhages are due to uterine atony.

A cardiotocograph shows type 1 variable decelerations. What is the cause of


this feature?

Your answer:

Umbilical cord compression

Correct answer:

Umbilical cord compression

The answer is umbilical cord compression. Variable decelerations are due to


umbilical cord compression. Early decelerations are caused by compression of
the fetal head, and late decelerations are caused by fetal hypoxia that is
secondary to placental insufficiency. Fetal movements are related to
accelerations.

You are asked to assess a woman who is receiving magnesium sulphate


infusions for severe pre-eclampsia. They have passed only 5 ml urine in the last
2 hours. Tests demonstrate that their deep tendon reflexes are absent. What
other observation should you take?

Your answer:

Respiratory rate

Correct answer:

Respiratory rate

The answer is respiratory rate. This woman has signs of magnesium toxicity
(absent deep tendon reflexes), which is probably secondary to renal
impairment. Respiratory depression is a sign of increasing magnesium toxicity
is; therefore, the most appropriate follow-up would be to investigate the
woman's respiratory rate.

Polyglactin sutures are used extensively in surgical procedures, particularly to


ligate vessels. What are the key features of polyglactin sutures?

Your answer:

Braided, absorbable and synthetic


Correct answer:

Braided, absorbable and synthetic

The answer is braided, absorbable and synthetic. Polyglactin sutures are used
to ligate pedicles and close the uterus during a caesarean section. To achieve
this, the sutures are braided to prevent the knots from slipping.

A nulliparous woman presents with spontaneous rupture of membranes at 41


weeks of gestation. At 18:00h, her cervical dilatation is 3 cm. A further vaginal
examination at 22:00h reveals that her cervical dilatation is still 3 cm. At
02:10h, the fetus is in the occipitoposterior position and uterine activity is
present. What is the most appropriate action?

Your answer:

Commence intravenous oxytocin

Correct answer:

Commence intravenous oxytocin

The answer is commence intravenous oxytocin. The membranes have ruptured


already so amniotomy is not required. No progression has been made during
the first stage of labour. Therefore, the woman should be administered
intravenous oxytocin.
A 68-year-old woman presents with two episodes of postmenopausal bleeding.
She has a BMI of 23 and is otherwise healthy. An ultrasound shows that her
endometrial cavity is 4 mm thick, and an endometrial pipelle sample is taken
that yields a small volume of tissue. The pathology report suggests a neoplasm.
What is the most likely diagnosis?

Your answer:

Serous carcinoma

Correct answer:

Serous carcinoma

The answer is serous carcinoma. Serous carcinomas are typically seen in


postmenopausal women. The development of these carcinomas is not
associated with a raised BMI, diabetes or hypertension. The uterine tumour
can be very small (even in the presence of extra uterine spread), and therefore,
results from an ultrasound and even hysteroscopy can appear normal.

A 75-year-old woman presents with increased weight, loss of hair and a dislike
of cold weather. She was noted to have a high prolactin and normal
electrolytes. She is otherwise fit and well. What is the likely cause of the high
prolactin?

Your answer:

Hypothyroidism

Correct answer:
Hypothyroidism

The correct answer is hypothyroidism.

The midwife has asked you to review a 34-year-old multigravida who is 7 cm


dilated and progressing well. The CTG shows a variability of <5. She was given
pethidine when she was 6 cm dilated. A fetal blood sampling is done and
reveals a pH of 7.23. What will be your next line of management?

Your answer:

Reassure the woman

View correct answer

Correct answer:

Repeat FBS within 30 minutes

The correct answer is repeat FBS within 30 minutes.


A 30-year-old primigravida who is pregnant after IVF attends the Early
Pregnancy Unit with severe right sided abdominal pain. An ultrasound scan
shows an empty uterus. β-hCG levels are 5,500 iu/L. What is the appropriate
course of action?

Your answer:

Expectant management, evaluate after 48 hours with repeat hCG levels

View correct answer

Correct answer:

Laparoscopy and salpingectomy if there is an ectopic pregnancy

The correct answer is laparoscopy and salpingectomy if there is an ectopic


pregnancy.

A 54-year-old woman presents to her GP with malaise, a history of recent


weight loss and bloating. On examination the GP notes a large pelvic mass. He
sends her for an urgent abdominal ultrasound scan and initiates a 2-week-wait
referral. This woman is discussed at an MDT meeting and it is decided to
proceed with a laparatomy. During the laparotomy, a full examination is
performed to look for possible metastatic spread. If this cancer is presumed to
be ovarian, which lymph nodes would you expect the disease to drain to first?

Your answer:
Para-aortic lymph nodes

Correct answer:

Para-aortic lymph nodes

The best answer is para-aortic lymph nodes.

A 28-year-old primiparous woman is induced at 39+2 due to a raised blood


pressure and significant proteinuria. She progresses well to full dilatation but
after pushing for 2 hours there is no vertex visible. A decision is made for an
instrumental delivery, and the baby is delivered via an assisted vaginal delivery
in theatre. The episiotomy extends and the tear involves approximately 60% of
the external anal sphincter. The internal anal sphincter and anal mucosa are
intact. How would you classify this perineal trauma?

Your answer:

3b degree tear

Correct answer:

3b degree tear
The best answer is a 3b degree tear. Note that the risk of perineal tearing after
instrumental delivery is 7% for forceps and 4% for ventouse.

A 28-year-old primiparous woman is induced at 39+2 due to a raised blood


pressure and significant proteinuria. She progresses well to full dilatation but
after pushing for 2 hours there is no vertex visible. On examination, the head is
felt to be in an occiput anterior position at +1 station. A decision is made for an
instrumental delivery, and the baby is delivered via assisted vaginal delivery in
theatre. For a ventouse delivery to be successful, the ventouse cup needs to be
applied to the flexion point of the fetal head. Where on the fetal head is the
flexion point located?

Your answer:

On the sagittal suture line, approximately 3 cm anterior (in front) of the


posterior fontanelle

Correct answer:

On the sagittal suture line, approximately 3 cm anterior (in front) of the


posterior fontanelle

The best answer is on the sagittal suture line, approximately 3 cm anterior (in
front) of the posterior fontanelle.
A woman with a previous caesarean section arrives at term to your unit in
spontaneous labour. The midwife reports the cervix is 6 cm dilated, there is a
longitudinal lie and the vertex is well applied to the cervix. The woman is
contracting three times in 10 minutes. The contractions stop and there is a
fetal bradycardia. What is the most likely diagnosis?

Your answer:

Scar rupture

Correct answer:

Scar rupture

The best answer is scar rupture as the fetus is compromised.

A woman with a previous caesarean section arrives at term to your unit in


spontaneous labour. The midwife reports the cervix is 6 cm dilated, there is a
longitudinal lie and the vertex is well applied to the cervix. The woman is
contracting three times in 10 minutes. You are asked to assess the woman by
the registrar on call as he is in theatre. What initial management would you
suggest?

Your answer:

You immediately perform a fetal blood sample


View correct answer

Correct answer:

You suggest continuous fetal monitoring (CFM) and vaginal assessment 2 hours
following the last VE

The best answer is CFM and vaginal assessment 2 hours following the last VE.

A 15-year-old girl visits her GP requesting a method of contraception. When


taking her history, you discover a personal history of thromboembolic events.
She has had three sexual partners in 5 months. She also complains of
dysmenorrhea. You advise on contraception being mindful of which medical
law?

Your answer:

Fraser

Correct answer:

Fraser

The correct answer is Fraser. This law refers specifically to contraception.


Add/Edit reflective notes

< Previous pageNext page >

Royal College of Obstetricians and Gynaecologists

© 2018

Registered charity no. 213280

27 Sussex Place
Regent's Park
London NW1 4RG
UK

Tel +44 20 7772 6200

Fax +44 20 7723 0575

Contact Us

Accessibility

Privacy & cookie policy


Terms & conditions

facebook

twitter

linkedin

google

youtube

vimeo


Skip to main content

Toggle navigation

Search StratOG

Home

Exam preparation

Part 1 MRCOG

SBAs for Part 1 MRCOG online resource

Clinical management

Tutorial history

Progress

100% complete
Submit

Clinical management

Assessment History

Total Attempts:5

Highest Score:100 %

View last resultsView highest resultRetake quiz

Your result86%

A 16-year-old woman presents with secondary amenorrhoea. She is healthy,


with no past medical history. Her BMI is 17. What is the most appropriate
initial investigation?

Your answer:

Follicle-stimulating hormone measurement

Correct answer:

Follicle-stimulating hormone measurement


The answer is follicle-stimulating hormone measurement. The likely diagnosis
is amenorrhoea caused by weight loss, so follicle-stimulating hormone
measurements would be appropriate. Since she is clinically euthyroid, thyroid
function tests would be of limited value.

A 55-year-old woman presents to the clinic enquiring about the use of


hormone replacement therapy (HRT). She had a hysterectomy 8 years ago for
fibroids. She has no contraindications or other past medical history, except a
strong family history of osteoporosis. Her main symptoms are hot flushes and
vaginal dryness. What would be your first treatment option?

Your answer:

Estrogen-only HRT

Correct answer:

Estrogen-only HRT

The answer is estrogen-only HRT. HRT provides the most effective method of
treating climacteric hot flushes and vaginal dryness, as well as postmenopausal
osteoporosis. Estrogen-only HRT is appropriate following a hysterectomy, as
the risk of endometrial carcinoma is not present.

An 18-year-old woman presents to an early pregnancy unit with light vaginal


bleeding after 10 weeks of amenorrhea. She had taken a pregnancy test 4
weeks ago that had tested positive. A transvaginal ultrasound scan showed an
irregular gestation sac with no fetal pole. What is the likely diagnosis?

Your answer:
Anembryonic pregnancy

Correct answer:

Anembryonic pregnancy

The answer is anembryonic pregnancy. Threatened miscarriage refers to


vaginal bleeding in the presence of a viable pregnancy; however, this
pregnancy is not viable. There are products of conception (gestation sac) that
are visible on the scan so this is an incomplete miscarriage, which is usually
associated with heavy vaginal bleeding. In this case, the woman is considered
to be pregnant, although no embryonic tissue is present. Therefore, the most
likely diagnosis is anembryonic pregnancy.

A primigravid woman presents in spontaneous labour at 39 weeks of gestation.


At 18:00h, her cervical dilatation is 6 cm. A further vaginal examination at
22:00h reveals that cervical dilatation is still at 6 cm. At 02:10h, the fetus is in
the occipitoposterior position and uterine activity is present. What is the most
appropriate action?

Your answer:

Amniotomy

Correct answer:

Amniotomy
The answer is amniotomy. This case demonstrates slow/no progression during
the first stage of labour and malpositioning of the fetus. The most appropriate
initial intervention would be an amniotomy (artificial rupture of the
membranes).

A 17-year-old woman presents to the sexual health clinic with vulval ulceration
and difficulty in passing urine. She is sexually active and has had unprotected
intercourse with her new boyfriend. She takes the combined oral
contraceptive pill. What is the most likely diagnosis?

Your answer:

Herpes simplex virus

Correct answer:

Herpes simplex virus

The answer is herpes simplex virus. A painful genital ulcer has developed in a
sexually active patient who has not used barrier contraception. Syphilis is
usually associated with the presence of a painless ulcer. Candida is not a
sexually transmitted disease and rarely presents with ulceration. Human
papillomavirus causes warts, and herpes varicella zoster causes chicken pox
and shingles.

Whilst you are attending to a woman in the antenatal ward, the


woman collapses and becomes unresponsive. You open their airway but they
are not breathing. What should you do next?

Your answer:
Get help

Correct answer:

Get help

The answer is get help. Basic and Advanced Life Support guidelines highlight
the need to get help if a patient has collapsed and is unresponsive. You should
then place the patient in left lateral tilt and commence cardiac compressions at
a rate of 30:2. Artificial ventilation is not mandatory in the ALS guidelines.

You answer an emergency call for a postpartum haemorrhage. The midwife


estimates that the woman has lost approximately 1 l of blood. What should
you be your first action?

Your answer:

Assess the woman’s airway, breathing and circulation and administer oxygen
at a rate of 15 l/min

Correct answer:

Assess the woman’s airway, breathing and circulation and administer oxygen
at a rate of 15 l/min

The answer is assess the woman’s airway, breathing and circulation and
administer oxygen at a rate of 15 l/min. In an acute emergency, one should
always assess the airway, breathing and circulation before addressing the
secondary treatment. In practice, this may be talking to the woman and to see
if they respond.

You answer an emergency call for a postpartum haemorrhage. The midwife


estimates that the woman has lost approximately 500 ml of blood. What is the
most likely cause of the bleeding?

Your answer:

Uterine atony

Correct answer:

Uterine atony

The answer is uterine atony. Approximately 70% of all postpartum


haemorrhages are due to uterine atony.

A cardiotocograph shows type 1 variable decelerations. What is the cause of


this feature?

Your answer:

Umbilical cord compression

Correct answer:

Umbilical cord compression


The answer is umbilical cord compression. Variable decelerations are due to
umbilical cord compression. Early decelerations are caused by compression of
the fetal head, and late decelerations are caused by fetal hypoxia that is
secondary to placental insufficiency. Fetal movements are related to
accelerations.

You are asked to assess a woman who is receiving magnesium sulphate


infusions for severe pre-eclampsia. They have passed only 5 ml urine in the last
2 hours. Tests demonstrate that their deep tendon reflexes are absent. What
other observation should you take?

Your answer:

Respiratory rate

Correct answer:

Respiratory rate

The answer is respiratory rate. This woman has signs of magnesium toxicity
(absent deep tendon reflexes), which is probably secondary to renal
impairment. Respiratory depression is a sign of increasing magnesium toxicity
is; therefore, the most appropriate follow-up would be to investigate the
woman's respiratory rate.

Polyglactin sutures are used extensively in surgical procedures, particularly to


ligate vessels. What are the key features of polyglactin sutures?

Your answer:
Braided, absorbable and synthetic

Correct answer:

Braided, absorbable and synthetic

The answer is braided, absorbable and synthetic. Polyglactin sutures are used
to ligate pedicles and close the uterus during a caesarean section. To achieve
this, the sutures are braided to prevent the knots from slipping.

A nulliparous woman presents with spontaneous rupture of membranes at 41


weeks of gestation. At 18:00h, her cervical dilatation is 3 cm. A further vaginal
examination at 22:00h reveals that her cervical dilatation is still 3 cm. At
02:10h, the fetus is in the occipitoposterior position and uterine activity is
present. What is the most appropriate action?

Your answer:

Commence intravenous oxytocin

Correct answer:

Commence intravenous oxytocin

The answer is commence intravenous oxytocin. The membranes have ruptured


already so amniotomy is not required. No progression has been made during
the first stage of labour. Therefore, the woman should be administered
intravenous oxytocin.
A 68-year-old woman presents with two episodes of postmenopausal bleeding.
She has a BMI of 23 and is otherwise healthy. An ultrasound shows that her
endometrial cavity is 4 mm thick, and an endometrial pipelle sample is taken
that yields a small volume of tissue. The pathology report suggests a neoplasm.
What is the most likely diagnosis?

Your answer:

Serous carcinoma

Correct answer:

Serous carcinoma

The answer is serous carcinoma. Serous carcinomas are typically seen in


postmenopausal women. The development of these carcinomas is not
associated with a raised BMI, diabetes or hypertension. The uterine tumour
can be very small (even in the presence of extra uterine spread), and therefore,
results from an ultrasound and even hysteroscopy can appear normal.

A 75-year-old woman presents with increased weight, loss of hair and a dislike
of cold weather. She was noted to have a high prolactin and normal
electrolytes. She is otherwise fit and well. What is the likely cause of the high
prolactin?

Your answer:

Hypothyroidism
Correct answer:

Hypothyroidism

The correct answer is hypothyroidism.

The midwife has asked you to review a 34-year-old multigravida who is 7 cm


dilated and progressing well. The CTG shows a variability of <5. She was given
pethidine when she was 6 cm dilated. A fetal blood sampling is done and
reveals a pH of 7.23. What will be your next line of management?

Your answer:

Reassure the woman

View correct answer

Correct answer:

Repeat FBS within 30 minutes

The correct answer is repeat FBS within 30 minutes.


A 30-year-old primigravida who is pregnant after IVF attends the Early
Pregnancy Unit with severe right sided abdominal pain. An ultrasound scan
shows an empty uterus. β-hCG levels are 5,500 iu/L. What is the appropriate
course of action?

Your answer:

Expectant management, evaluate after 48 hours with repeat hCG levels

View correct answer

Correct answer:

Laparoscopy and salpingectomy if there is an ectopic pregnancy

The correct answer is laparoscopy and salpingectomy if there is an ectopic


pregnancy.

A 54-year-old woman presents to her GP with malaise, a history of recent


weight loss and bloating. On examination the GP notes a large pelvic mass. He
sends her for an urgent abdominal ultrasound scan and initiates a 2-week-wait
referral. This woman is discussed at an MDT meeting and it is decided to
proceed with a laparatomy. During the laparotomy, a full examination is
performed to look for possible metastatic spread. If this cancer is presumed to
be ovarian, which lymph nodes would you expect the disease to drain to first?

Your answer:
Para-aortic lymph nodes

Correct answer:

Para-aortic lymph nodes

The best answer is para-aortic lymph nodes.

A 28-year-old primiparous woman is induced at 39+2 due to a raised blood


pressure and significant proteinuria. She progresses well to full dilatation but
after pushing for 2 hours there is no vertex visible. A decision is made for an
instrumental delivery, and the baby is delivered via an assisted vaginal delivery
in theatre. The episiotomy extends and the tear involves approximately 60% of
the external anal sphincter. The internal anal sphincter and anal mucosa are
intact. How would you classify this perineal trauma?

Your answer:

3b degree tear

Correct answer:

3b degree tear
The best answer is a 3b degree tear. Note that the risk of perineal tearing after
instrumental delivery is 7% for forceps and 4% for ventouse.

A 28-year-old primiparous woman is induced at 39+2 due to a raised blood


pressure and significant proteinuria. She progresses well to full dilatation but
after pushing for 2 hours there is no vertex visible. On examination, the head is
felt to be in an occiput anterior position at +1 station. A decision is made for an
instrumental delivery, and the baby is delivered via assisted vaginal delivery in
theatre. For a ventouse delivery to be successful, the ventouse cup needs to be
applied to the flexion point of the fetal head. Where on the fetal head is the
flexion point located?

Your answer:

On the sagittal suture line, approximately 3 cm anterior (in front) of the


posterior fontanelle

Correct answer:

On the sagittal suture line, approximately 3 cm anterior (in front) of the


posterior fontanelle

The best answer is on the sagittal suture line, approximately 3 cm anterior (in
front) of the posterior fontanelle.
A woman with a previous caesarean section arrives at term to your unit in
spontaneous labour. The midwife reports the cervix is 6 cm dilated, there is a
longitudinal lie and the vertex is well applied to the cervix. The woman is
contracting three times in 10 minutes. The contractions stop and there is a
fetal bradycardia. What is the most likely diagnosis?

Your answer:

Scar rupture

Correct answer:

Scar rupture

The best answer is scar rupture as the fetus is compromised.

A woman with a previous caesarean section arrives at term to your unit in


spontaneous labour. The midwife reports the cervix is 6 cm dilated, there is a
longitudinal lie and the vertex is well applied to the cervix. The woman is
contracting three times in 10 minutes. You are asked to assess the woman by
the registrar on call as he is in theatre. What initial management would you
suggest?

Your answer:

You immediately perform a fetal blood sample


View correct answer

Correct answer:

You suggest continuous fetal monitoring (CFM) and vaginal assessment 2 hours
following the last VE

The best answer is CFM and vaginal assessment 2 hours following the last VE.

A 15-year-old girl visits her GP requesting a method of contraception. When


taking her history, you discover a personal history of thromboembolic events.
She has had three sexual partners in 5 months. She also complains of
dysmenorrhea. You advise on contraception being mindful of which medical
law?

Your answer:

Fraser

Correct answer:

Fraser

The correct answer is Fraser. This law refers specifically to contraception.


Add/Edit reflective notes

< Previous pageNext page >

Royal College of Obstetricians and Gynaecologists

© 2018

Registered charity no. 213280

27 Sussex Place
Regent's Park
London NW1 4RG
UK

Tel +44 20 7772 6200

Fax +44 20 7723 0575

Contact Us

Accessibility

Privacy & cookie policy


Terms & conditions

facebook

twitter

linkedin

google

youtube

vimeo

You might also like